Quiz-summary
0 of 29 questions completed
Questions:
- 1
- 2
- 3
- 4
- 5
- 6
- 7
- 8
- 9
- 10
- 11
- 12
- 13
- 14
- 15
- 16
- 17
- 18
- 19
- 20
- 21
- 22
- 23
- 24
- 25
- 26
- 27
- 28
- 29
Information
Premium Practice Questions
You have already completed the quiz before. Hence you can not start it again.
Quiz is loading...
You must sign in or sign up to start the quiz.
You have to finish following quiz, to start this quiz:
Results
0 of 29 questions answered correctly
Your time:
Time has elapsed
Categories
- Not categorized 0%
- 1
- 2
- 3
- 4
- 5
- 6
- 7
- 8
- 9
- 10
- 11
- 12
- 13
- 14
- 15
- 16
- 17
- 18
- 19
- 20
- 21
- 22
- 23
- 24
- 25
- 26
- 27
- 28
- 29
- Answered
- Review
-
Question 1 of 29
1. Question
A 72-year-old male presents with a new, firm, erythematous papule on his sun-exposed left forearm. A shave biopsy is performed. Histologic examination reveals a well-demarcated endophytic proliferation of atypical squamous cells extending into the superficial dermis. The neoplastic cells exhibit moderate nuclear pleomorphism, hyperchromasia, and scattered mitotic figures. Foci of keratinization with the formation of keratin pearls are evident within the tumor nests. Immunohistochemical stains reveal strong nuclear and cytoplasmic positivity for p63 and strong cytoplasmic positivity for CK5/6 in the atypical cells. The tumor cells are negative for S100 and SOX10. Considering the clinical presentation and the histopathological and immunohistochemical findings, what is the most likely diagnosis for this American Board of Pathology – Subspecialty in Dermatopathology University candidate to consider?
Correct
The scenario describes a patient with a history of chronic sun exposure and the development of a new, indurated papule on the forearm. Histopathological examination reveals a proliferation of atypical keratinocytes with significant nuclear pleomorphism, hyperchromasia, and keratin pearl formation, invading the dermis. Immunohistochemical staining demonstrates strong positivity for p63 and CK5/6, and negativity for S100 and SOX10. The differential diagnosis for such a lesion includes squamous cell carcinoma, keratoacanthoma, and potentially a poorly differentiated adnexal neoplasm. However, the combination of dermal invasion, significant atypia, and the characteristic keratin pearl formation strongly favors squamous cell carcinoma. Keratoacanthoma, while showing rapid growth and keratin plugging, typically exhibits a more benign cytologic appearance and a characteristic “collar of basaloid cells” at the base of the invaginated keratin plug, which is not explicitly described here. Poorly differentiated adnexal neoplasms would likely show different immunohistochemical profiles, such as positivity for markers like EMA or specific cytokeratins related to follicular or glandular differentiation. The positive staining for p63 and CK5/6 confirms an epithelial origin, specifically from the squamous lineage, and their strong expression in the atypical cells, coupled with the absence of melanocytic markers (S100, SOX10), solidifies the diagnosis of squamous cell carcinoma. The absence of specific features that would definitively point to keratoacanthoma, and the lack of evidence for adnexal differentiation, makes squamous cell carcinoma the most accurate interpretation of the provided histopathological and immunohistochemical findings.
Incorrect
The scenario describes a patient with a history of chronic sun exposure and the development of a new, indurated papule on the forearm. Histopathological examination reveals a proliferation of atypical keratinocytes with significant nuclear pleomorphism, hyperchromasia, and keratin pearl formation, invading the dermis. Immunohistochemical staining demonstrates strong positivity for p63 and CK5/6, and negativity for S100 and SOX10. The differential diagnosis for such a lesion includes squamous cell carcinoma, keratoacanthoma, and potentially a poorly differentiated adnexal neoplasm. However, the combination of dermal invasion, significant atypia, and the characteristic keratin pearl formation strongly favors squamous cell carcinoma. Keratoacanthoma, while showing rapid growth and keratin plugging, typically exhibits a more benign cytologic appearance and a characteristic “collar of basaloid cells” at the base of the invaginated keratin plug, which is not explicitly described here. Poorly differentiated adnexal neoplasms would likely show different immunohistochemical profiles, such as positivity for markers like EMA or specific cytokeratins related to follicular or glandular differentiation. The positive staining for p63 and CK5/6 confirms an epithelial origin, specifically from the squamous lineage, and their strong expression in the atypical cells, coupled with the absence of melanocytic markers (S100, SOX10), solidifies the diagnosis of squamous cell carcinoma. The absence of specific features that would definitively point to keratoacanthoma, and the lack of evidence for adnexal differentiation, makes squamous cell carcinoma the most accurate interpretation of the provided histopathological and immunohistochemical findings.
-
Question 2 of 29
2. Question
A 68-year-old male presents with multiple, asymptomatic, erythematous papules and plaques on his trunk and extremities, present for several months. A skin biopsy is performed. Histopathological examination reveals a dense, diffuse infiltrate of small to medium-sized lymphoid cells in the superficial and deep dermis, with some nodular aggregates. The cells exhibit mild nuclear irregularity and scant cytoplasm. Immunohistochemical stains are performed, revealing the lymphoid population to be positive for CD20, CD5, and Cyclin D1, with a Ki-67 proliferation index of approximately 15%. Considering the clinical presentation, histological findings, and immunophenotype, which specific subtype of cutaneous B-cell lymphoma is most strongly suggested by this constellation of findings for the American Board of Pathology – Subspecialty in Dermatopathology University curriculum?
Correct
The question assesses the understanding of the interplay between specific immunohistochemical markers and the diagnostic interpretation of cutaneous lymphoid proliferations, a core competency in dermatopathology. The scenario describes a biopsy with features suggestive of a B-cell lymphoma, specifically mentioning atypical lymphoid cells with a CD20+ and CD5+ immunophenotype. In the context of cutaneous lymphoid proliferations, a CD20+, CD5+ profile in B-cells is highly characteristic of **mantle cell lymphoma (MCL)**. MCL is a type of non-Hodgkin lymphoma that can manifest in the skin, often presenting as multiple erythematous papules, plaques, or nodules. Histologically, it typically shows a dense infiltrate of small to medium-sized lymphoid cells with irregular nuclei and scant cytoplasm. The immunophenotype is crucial for differentiating MCL from other B-cell lymphomas that may involve the skin. While CD5 can be expressed by other B-cell subsets (like chronic lymphocytic leukemia/small lymphocytic lymphoma), its co-expression with CD20 in the context of a cutaneous lymphoid infiltrate, particularly with the described morphology, strongly points towards MCL. Other options are less likely given the provided immunophenotype. For instance, follicular lymphoma typically expresses CD10 and Bcl-6, and is CD5-. Extranodal marginal zone lymphoma (MALT lymphoma) can be CD5-, and while it can involve the skin, the CD5 positivity here makes it less probable as the primary diagnosis. Cutaneous B-cell lymphomas, NOS, is a broad category, but MCL has a more specific immunophenotypic signature. Therefore, the most precise diagnosis based on the provided information is mantle cell lymphoma.
Incorrect
The question assesses the understanding of the interplay between specific immunohistochemical markers and the diagnostic interpretation of cutaneous lymphoid proliferations, a core competency in dermatopathology. The scenario describes a biopsy with features suggestive of a B-cell lymphoma, specifically mentioning atypical lymphoid cells with a CD20+ and CD5+ immunophenotype. In the context of cutaneous lymphoid proliferations, a CD20+, CD5+ profile in B-cells is highly characteristic of **mantle cell lymphoma (MCL)**. MCL is a type of non-Hodgkin lymphoma that can manifest in the skin, often presenting as multiple erythematous papules, plaques, or nodules. Histologically, it typically shows a dense infiltrate of small to medium-sized lymphoid cells with irregular nuclei and scant cytoplasm. The immunophenotype is crucial for differentiating MCL from other B-cell lymphomas that may involve the skin. While CD5 can be expressed by other B-cell subsets (like chronic lymphocytic leukemia/small lymphocytic lymphoma), its co-expression with CD20 in the context of a cutaneous lymphoid infiltrate, particularly with the described morphology, strongly points towards MCL. Other options are less likely given the provided immunophenotype. For instance, follicular lymphoma typically expresses CD10 and Bcl-6, and is CD5-. Extranodal marginal zone lymphoma (MALT lymphoma) can be CD5-, and while it can involve the skin, the CD5 positivity here makes it less probable as the primary diagnosis. Cutaneous B-cell lymphomas, NOS, is a broad category, but MCL has a more specific immunophenotypic signature. Therefore, the most precise diagnosis based on the provided information is mantle cell lymphoma.
-
Question 3 of 29
3. Question
Consider a challenging case presented to a dermatopathology fellow at the American Board of Pathology – Subspecialty in Dermatopathology University. The biopsy reveals a dermal infiltrate with atypical cells exhibiting pleomorphism and occasional mitotic figures. Initial H&E staining raises suspicion for a cutaneous melanoma. To further refine the diagnosis, immunohistochemical stains are performed. The tumor cells demonstrate strong and diffuse positivity for Melan-A and Tyrosinase, but only focal, weak positivity for S100. Crucially, the same tumor cells also show distinct cytoplasmic positivity for CK7. Based on these findings, what is the most likely interpretation of the cutaneous neoplasm?
Correct
The question probes the understanding of immunohistochemical markers in differentiating challenging cutaneous neoplasms, specifically focusing on the distinction between a primary cutaneous melanoma and a metastatic melanoma from an extracutaneous primary site. In this scenario, a lesion exhibits features suggestive of melanoma, and the pathologist needs to determine its origin. Melan-A (MART-1) and Tyrosinase are considered relatively sensitive and specific markers for melanocytic differentiation, often expressed in both primary cutaneous melanomas and metastatic melanomas of cutaneous origin. However, their expression can sometimes be seen in other melanocytic lesions. S100 protein is a more broadly expressed marker of neural crest-derived cells, including melanocytes, but it also stains other cell types, such as some nerve sheath tumors and adipocytes, making it less specific for definitive melanoma diagnosis alone. Conversely, CK7 is a keratin marker typically expressed in epithelial cells and is frequently positive in adenocarcinomas, which are common sources of cutaneous metastases. Therefore, the presence of CK7 positivity, especially in conjunction with negative or equivocal staining for melanocytic markers, strongly suggests a metastatic adenocarcinoma rather than a primary cutaneous melanoma. The absence of strong, diffuse Melan-A and Tyrosinase, coupled with positive CK7, points towards an epithelial origin for the metastasis. This diagnostic approach is crucial in dermatopathology at the American Board of Pathology – Subspecialty in Dermatopathology University, where accurate tumor origin determination impacts patient management and prognosis. Understanding the differential expression patterns of these markers is fundamental for distinguishing primary cutaneous neoplasms from metastatic disease, a critical skill for aspiring dermatopathologists.
Incorrect
The question probes the understanding of immunohistochemical markers in differentiating challenging cutaneous neoplasms, specifically focusing on the distinction between a primary cutaneous melanoma and a metastatic melanoma from an extracutaneous primary site. In this scenario, a lesion exhibits features suggestive of melanoma, and the pathologist needs to determine its origin. Melan-A (MART-1) and Tyrosinase are considered relatively sensitive and specific markers for melanocytic differentiation, often expressed in both primary cutaneous melanomas and metastatic melanomas of cutaneous origin. However, their expression can sometimes be seen in other melanocytic lesions. S100 protein is a more broadly expressed marker of neural crest-derived cells, including melanocytes, but it also stains other cell types, such as some nerve sheath tumors and adipocytes, making it less specific for definitive melanoma diagnosis alone. Conversely, CK7 is a keratin marker typically expressed in epithelial cells and is frequently positive in adenocarcinomas, which are common sources of cutaneous metastases. Therefore, the presence of CK7 positivity, especially in conjunction with negative or equivocal staining for melanocytic markers, strongly suggests a metastatic adenocarcinoma rather than a primary cutaneous melanoma. The absence of strong, diffuse Melan-A and Tyrosinase, coupled with positive CK7, points towards an epithelial origin for the metastasis. This diagnostic approach is crucial in dermatopathology at the American Board of Pathology – Subspecialty in Dermatopathology University, where accurate tumor origin determination impacts patient management and prognosis. Understanding the differential expression patterns of these markers is fundamental for distinguishing primary cutaneous neoplasms from metastatic disease, a critical skill for aspiring dermatopathologists.
-
Question 4 of 29
4. Question
In the context of differentiating a high-grade sebaceous carcinoma from a basal cell carcinoma exhibiting focal sebaceous metaplasia, which immunohistochemical panel would provide the most discriminative diagnostic utility for a candidate at the American Board of Pathology – Subspecialty in Dermatopathology University, considering the typical expression patterns of these markers in adnexal neoplasms?
Correct
The question probes the nuanced understanding of immunohistochemical panel selection for differentiating challenging adnexal neoplasms, specifically focusing on the distinction between a sebaceous carcinoma and a basal cell carcinoma with sebaceous differentiation. A sebaceous carcinoma is a malignant neoplasm of sebaceous glands, characterized by atypical sebaceous cells. Immunohistochemistry plays a crucial role in confirming sebaceous differentiation and assessing malignancy. For sebaceous carcinoma, key markers include: * **PAX-8:** Typically positive in sebaceous carcinomas, reflecting their glandular origin. * **Androgen Receptor (AR):** Often positive in sebaceous carcinomas, as sebaceous glands are androgen-sensitive. * **CD34:** Can be positive in the stroma surrounding sebaceous carcinoma, but its direct positivity in the tumor cells is less consistent than in other adnexal tumors. However, it can be helpful in assessing vascularity and stromal changes. * **CK7:** Generally positive in sebaceous carcinomas, indicating ductal differentiation. * **Ki-67:** Elevated proliferation index is indicative of malignancy. For basal cell carcinoma with sebaceous differentiation, the primary markers would reflect the basaloid nature of the tumor: * **Ber-EP4:** Strongly and diffusely positive in basal cell carcinomas, highlighting the epithelial nature of the tumor cells. * **CK20:** Typically negative in basal cell carcinomas. * **CK7:** Can be positive in some basal cell carcinomas, but less consistently than in sebaceous carcinoma. * **PAX-8:** Generally negative in basal cell carcinomas. * **Androgen Receptor (AR):** Typically negative in basal cell carcinomas. Considering the differential diagnosis, a panel that effectively distinguishes between these two entities would include markers that are characteristically positive in one and negative or variably positive in the other. PAX-8 and Androgen Receptor are particularly useful for identifying sebaceous differentiation and are typically positive in sebaceous carcinoma, while basal cell carcinoma, even with sebaceous elements, would likely be negative for these markers and positive for Ber-EP4. CK7 can be positive in both but is often more strongly and diffusely positive in sebaceous carcinoma. CD34 is more relevant for assessing stromal changes or other adnexal tumors like hidradenomas. Therefore, a panel including PAX-8, Androgen Receptor, and CK7, along with Ber-EP4 as a control for basaloid differentiation, would be most effective. The question asks for the *most appropriate* panel to *confirm* sebaceous differentiation and assess malignancy. A panel that highlights the sebaceous component and its potential for atypia is paramount. PAX-8 and Androgen Receptor are highly specific for sebaceous differentiation. CK7 supports glandular origin. While Ber-EP4 is crucial for BCC, its absence or weak expression in the context of PAX-8 and AR positivity would further support sebaceous carcinoma. Therefore, a panel focusing on sebaceous differentiation markers is key. The correct approach involves selecting markers that highlight the sebaceous lineage and potential for malignancy. PAX-8 and Androgen Receptor are strong indicators of sebaceous differentiation. CK7 is also often positive in sebaceous neoplasms. While Ber-EP4 is a marker for basal cell carcinoma, its inclusion in a panel to *confirm* sebaceous differentiation is less direct than markers specifically targeting sebaceous elements. CD34 is more relevant for vascularity or other adnexal tumors. Therefore, a panel comprising PAX-8, Androgen Receptor, and CK7 would be most effective in confirming sebaceous differentiation and assessing the neoplastic nature of the cells, distinguishing it from other possibilities like basal cell carcinoma with sebaceous elements.
Incorrect
The question probes the nuanced understanding of immunohistochemical panel selection for differentiating challenging adnexal neoplasms, specifically focusing on the distinction between a sebaceous carcinoma and a basal cell carcinoma with sebaceous differentiation. A sebaceous carcinoma is a malignant neoplasm of sebaceous glands, characterized by atypical sebaceous cells. Immunohistochemistry plays a crucial role in confirming sebaceous differentiation and assessing malignancy. For sebaceous carcinoma, key markers include: * **PAX-8:** Typically positive in sebaceous carcinomas, reflecting their glandular origin. * **Androgen Receptor (AR):** Often positive in sebaceous carcinomas, as sebaceous glands are androgen-sensitive. * **CD34:** Can be positive in the stroma surrounding sebaceous carcinoma, but its direct positivity in the tumor cells is less consistent than in other adnexal tumors. However, it can be helpful in assessing vascularity and stromal changes. * **CK7:** Generally positive in sebaceous carcinomas, indicating ductal differentiation. * **Ki-67:** Elevated proliferation index is indicative of malignancy. For basal cell carcinoma with sebaceous differentiation, the primary markers would reflect the basaloid nature of the tumor: * **Ber-EP4:** Strongly and diffusely positive in basal cell carcinomas, highlighting the epithelial nature of the tumor cells. * **CK20:** Typically negative in basal cell carcinomas. * **CK7:** Can be positive in some basal cell carcinomas, but less consistently than in sebaceous carcinoma. * **PAX-8:** Generally negative in basal cell carcinomas. * **Androgen Receptor (AR):** Typically negative in basal cell carcinomas. Considering the differential diagnosis, a panel that effectively distinguishes between these two entities would include markers that are characteristically positive in one and negative or variably positive in the other. PAX-8 and Androgen Receptor are particularly useful for identifying sebaceous differentiation and are typically positive in sebaceous carcinoma, while basal cell carcinoma, even with sebaceous elements, would likely be negative for these markers and positive for Ber-EP4. CK7 can be positive in both but is often more strongly and diffusely positive in sebaceous carcinoma. CD34 is more relevant for assessing stromal changes or other adnexal tumors like hidradenomas. Therefore, a panel including PAX-8, Androgen Receptor, and CK7, along with Ber-EP4 as a control for basaloid differentiation, would be most effective. The question asks for the *most appropriate* panel to *confirm* sebaceous differentiation and assess malignancy. A panel that highlights the sebaceous component and its potential for atypia is paramount. PAX-8 and Androgen Receptor are highly specific for sebaceous differentiation. CK7 supports glandular origin. While Ber-EP4 is crucial for BCC, its absence or weak expression in the context of PAX-8 and AR positivity would further support sebaceous carcinoma. Therefore, a panel focusing on sebaceous differentiation markers is key. The correct approach involves selecting markers that highlight the sebaceous lineage and potential for malignancy. PAX-8 and Androgen Receptor are strong indicators of sebaceous differentiation. CK7 is also often positive in sebaceous neoplasms. While Ber-EP4 is a marker for basal cell carcinoma, its inclusion in a panel to *confirm* sebaceous differentiation is less direct than markers specifically targeting sebaceous elements. CD34 is more relevant for vascularity or other adnexal tumors. Therefore, a panel comprising PAX-8, Androgen Receptor, and CK7 would be most effective in confirming sebaceous differentiation and assessing the neoplastic nature of the cells, distinguishing it from other possibilities like basal cell carcinoma with sebaceous elements.
-
Question 5 of 29
5. Question
A dermatopathology fellow at the American Board of Pathology – Subspecialty in Dermatopathology University is evaluating a challenging biopsy from the dorsal aspect of the forearm of a middle-aged patient. Histologically, the lesion demonstrates a dermal proliferation of atypical spindle cells arranged in fascicles and storiform patterns, with some pleomorphism and occasional mitotic figures. Immunohistochemical stains reveal focal positivity for S100 protein, but the cells are negative for cytokeratins (AE1/AE3) and CD34. Considering the differential diagnoses and the need for definitive lineage confirmation within the rigorous academic framework of the American Board of Pathology – Subspecialty in Dermatopathology University, which of the following ancillary immunohistochemical markers would be most crucial to further characterize this neoplasm and support a diagnosis of melanoma?
Correct
The question probes the understanding of the interplay between specific immunohistochemical markers and their diagnostic utility in differentiating challenging cutaneous neoplasms, a core competency for dermatopathology specialists at the American Board of Pathology – Subspecialty in Dermatopathology University. The scenario describes a biopsy showing atypical spindle cells with focal positivity for S100 and negativity for cytokeratins and CD34. This pattern is highly suggestive of a melanoma, specifically a desmoplastic melanoma or a spindle cell nevus, which can be diagnostically challenging. While S100 is a broad marker for melanocytic differentiation, its positivity in spindle cell lesions necessitates further investigation to confirm melanocytic lineage and rule out other spindle cell tumors. Cytokeratins are typically negative in melanocytic lesions and positive in epithelial neoplasms, thus their absence supports a non-epithelial origin. CD34 is often positive in dermatofibromas and some vascular tumors, which are important differential diagnoses for spindle cell lesions. Therefore, to definitively establish melanocytic differentiation and support a diagnosis of melanoma, particularly in the context of spindle cell morphology, additional markers are crucial. SOX10 is a highly sensitive and specific marker for melanocytic differentiation, including spindle cell melanomas, and is often used in conjunction with S100. Melan-A (MART-1) and Tyrosinase are also key melanocytic markers. Given the provided information, the most critical next step in confirming a melanocytic neoplasm, especially one with spindle cell morphology, would be to utilize markers that specifically confirm melanocytic lineage and can help differentiate between benign and malignant melanocytic proliferations. SOX10 is a master regulator of melanocyte development and is consistently expressed in melanomas, making it an excellent confirmatory marker in this context.
Incorrect
The question probes the understanding of the interplay between specific immunohistochemical markers and their diagnostic utility in differentiating challenging cutaneous neoplasms, a core competency for dermatopathology specialists at the American Board of Pathology – Subspecialty in Dermatopathology University. The scenario describes a biopsy showing atypical spindle cells with focal positivity for S100 and negativity for cytokeratins and CD34. This pattern is highly suggestive of a melanoma, specifically a desmoplastic melanoma or a spindle cell nevus, which can be diagnostically challenging. While S100 is a broad marker for melanocytic differentiation, its positivity in spindle cell lesions necessitates further investigation to confirm melanocytic lineage and rule out other spindle cell tumors. Cytokeratins are typically negative in melanocytic lesions and positive in epithelial neoplasms, thus their absence supports a non-epithelial origin. CD34 is often positive in dermatofibromas and some vascular tumors, which are important differential diagnoses for spindle cell lesions. Therefore, to definitively establish melanocytic differentiation and support a diagnosis of melanoma, particularly in the context of spindle cell morphology, additional markers are crucial. SOX10 is a highly sensitive and specific marker for melanocytic differentiation, including spindle cell melanomas, and is often used in conjunction with S100. Melan-A (MART-1) and Tyrosinase are also key melanocytic markers. Given the provided information, the most critical next step in confirming a melanocytic neoplasm, especially one with spindle cell morphology, would be to utilize markers that specifically confirm melanocytic lineage and can help differentiate between benign and malignant melanocytic proliferations. SOX10 is a master regulator of melanocyte development and is consistently expressed in melanomas, making it an excellent confirmatory marker in this context.
-
Question 6 of 29
6. Question
Consider a dermal nodule excised from the scalp of an elderly gentleman, presenting histologically with a well-circumscribed, intradermal proliferation of glandular structures and solid nests. The immunohistochemical panel applied to this specimen shows strong positivity for Cytokeratin 7 (CK7), positivity for Gross Cystic Disease Fluid Protein-15 (GCDFP-15), positivity for androgen receptor, and negativity for Carcinoembryonic Antigen (CEA). Based on this comprehensive panel, what is the most likely lineage of differentiation for this cutaneous neoplasm, a critical consideration for trainees at the American Board of Pathology – Subspecialty in Dermatopathology University?
Correct
The question probes the understanding of the interplay between specific immunohistochemical markers and the diagnostic classification of cutaneous adnexal neoplasms, particularly differentiating between eccrine and apocrine differentiation. In this scenario, a dermal nodule exhibits features suggestive of a sweat gland tumor. The immunohistochemical panel reveals positivity for CK7, GCDFP-15, and androgen receptor, with negativity for CEA. CK7 is a broad-spectrum keratin found in both eccrine and apocrine glands, thus it is not discriminatory on its own. GCDFP-15 (Gross Cystic Disease Fluid Protein-15) is a well-established marker strongly associated with apocrine differentiation, although it can occasionally be seen in some eccrine tumors. The androgen receptor is also frequently expressed in apocrine glands and apocrine tumors. CEA (Carcinoembryonic Antigen) is typically negative in apocrine tumors but can be positive in some eccrine neoplasms. Therefore, the combination of GCDFP-15 positivity and androgen receptor positivity, coupled with CEA negativity, strongly favors apocrine differentiation. The absence of specific eccrine markers like S-100 protein (which can be expressed in both but is more characteristic of eccrine ductal differentiation) or other eccrine-specific markers further solidifies this interpretation. The correct diagnostic conclusion, based on this panel, points towards an apocrine origin for the neoplasm. This understanding is crucial for accurate dermatopathology diagnosis and subsequent patient management, aligning with the rigorous standards of the American Board of Pathology – Subspecialty in Dermatopathology University, which emphasizes precise molecular and immunohistochemical correlation for nuanced diagnoses.
Incorrect
The question probes the understanding of the interplay between specific immunohistochemical markers and the diagnostic classification of cutaneous adnexal neoplasms, particularly differentiating between eccrine and apocrine differentiation. In this scenario, a dermal nodule exhibits features suggestive of a sweat gland tumor. The immunohistochemical panel reveals positivity for CK7, GCDFP-15, and androgen receptor, with negativity for CEA. CK7 is a broad-spectrum keratin found in both eccrine and apocrine glands, thus it is not discriminatory on its own. GCDFP-15 (Gross Cystic Disease Fluid Protein-15) is a well-established marker strongly associated with apocrine differentiation, although it can occasionally be seen in some eccrine tumors. The androgen receptor is also frequently expressed in apocrine glands and apocrine tumors. CEA (Carcinoembryonic Antigen) is typically negative in apocrine tumors but can be positive in some eccrine neoplasms. Therefore, the combination of GCDFP-15 positivity and androgen receptor positivity, coupled with CEA negativity, strongly favors apocrine differentiation. The absence of specific eccrine markers like S-100 protein (which can be expressed in both but is more characteristic of eccrine ductal differentiation) or other eccrine-specific markers further solidifies this interpretation. The correct diagnostic conclusion, based on this panel, points towards an apocrine origin for the neoplasm. This understanding is crucial for accurate dermatopathology diagnosis and subsequent patient management, aligning with the rigorous standards of the American Board of Pathology – Subspecialty in Dermatopathology University, which emphasizes precise molecular and immunohistochemical correlation for nuanced diagnoses.
-
Question 7 of 29
7. Question
A pathologist at the American Board of Pathology – Subspecialty in Dermatopathology University is evaluating a deeply invasive dermal nodule with atypical spindle cells and focal epithelioid morphology. The initial H&E staining raises suspicion for a melanoma. An immunohistochemical panel is performed, revealing positivity for S100, SOX10, and MITF. However, the differential diagnosis includes a metastatic carcinoma. To definitively differentiate between a primary cutaneous melanoma and a metastasis from a non-cutaneous epithelial malignancy, which of the following immunohistochemical findings would be most critical in supporting the latter diagnosis?
Correct
The question probes the understanding of immunohistochemical markers in differentiating challenging cutaneous neoplasms, specifically focusing on distinguishing between a primary cutaneous melanoma and a metastatic melanoma from a non-cutaneous primary. In this scenario, the immunohistochemical panel is crucial. Melanoma cells typically express S100 protein, SOX10, and MITF, which are generally positive in both primary cutaneous and metastatic melanomas. However, the key to differentiating the origin lies in markers that are either absent or aberrantly expressed in metastatic disease from non-cutaneous primaries. Cytokeratins (CKs) are intermediate filaments found in epithelial cells and are typically absent in melanomas. Their presence in a suspected melanoma metastasis would strongly suggest a non-melanoma epithelial origin, such as a carcinoma. Therefore, a positive CK stain in this context would point towards a metastatic carcinoma rather than a primary cutaneous melanoma or a melanoma metastatic from another site. Conversely, a negative CK stain would be consistent with a melanoma of cutaneous origin or a melanoma metastatic from another melanocytic site. The absence of CK expression is a critical negative finding that supports a melanocytic lineage.
Incorrect
The question probes the understanding of immunohistochemical markers in differentiating challenging cutaneous neoplasms, specifically focusing on distinguishing between a primary cutaneous melanoma and a metastatic melanoma from a non-cutaneous primary. In this scenario, the immunohistochemical panel is crucial. Melanoma cells typically express S100 protein, SOX10, and MITF, which are generally positive in both primary cutaneous and metastatic melanomas. However, the key to differentiating the origin lies in markers that are either absent or aberrantly expressed in metastatic disease from non-cutaneous primaries. Cytokeratins (CKs) are intermediate filaments found in epithelial cells and are typically absent in melanomas. Their presence in a suspected melanoma metastasis would strongly suggest a non-melanoma epithelial origin, such as a carcinoma. Therefore, a positive CK stain in this context would point towards a metastatic carcinoma rather than a primary cutaneous melanoma or a melanoma metastatic from another site. Conversely, a negative CK stain would be consistent with a melanoma of cutaneous origin or a melanoma metastatic from another melanocytic site. The absence of CK expression is a critical negative finding that supports a melanocytic lineage.
-
Question 8 of 29
8. Question
A 72-year-old male, a retired gardener with a history of extensive solar exposure, presents to the clinic with a new, firm, erythematous nodule on his left forearm that has doubled in size over the past three months. A shave biopsy is performed. Microscopic examination of the hematoxylin and eosin stained sections reveals a dermal-based, infiltrative neoplasm composed of atypical squamous cells with marked nuclear pleomorphism, irregular nuclear contours, prominent nucleoli, and frequent mitotic figures. The tumor exhibits areas of keratinization, including keratin pearls, and a significant surrounding desmoplastic stromal response. Immunohistochemical stains reveal diffuse and strong nuclear positivity for p53 in approximately 70% of the neoplastic cells, with a notable absence of suprabasal K10 expression. Perineural invasion is identified in multiple sections. Based on these findings, which of the following diagnoses most accurately characterizes this lesion in the context of dermatopathology training at American Board of Pathology – Subspecialty in Dermatopathology University?
Correct
The scenario describes a patient with a history of chronic sun exposure and a new, rapidly growing lesion on the forearm. Histopathological examination reveals a nodular proliferation of atypical keratinocytes with significant nuclear pleomorphism, hyperchromasia, and keratin pearl formation. Immunohistochemical staining demonstrates strong positivity for p53 in a significant proportion of the atypical cells, with a markedly reduced or absent expression of differentiation markers like K10 in the suprabasal layers. The presence of a prominent desmoplastic stromal reaction and perineural invasion further supports a high-grade malignancy. Considering the clinical presentation and the detailed histopathological and immunohistochemical findings, the most appropriate diagnosis that aligns with the aggressive features and the observed molecular marker expression is invasive squamous cell carcinoma with high-risk features. The p53 overexpression is a common indicator of DNA damage accumulation and is frequently associated with increased aggressiveness and poor prognosis in squamous cell carcinomas. The loss of K10 suggests a profound disruption of normal keratinocyte differentiation pathways. The desmoplastic stroma and perineural invasion are well-established indicators of increased metastatic potential and local recurrence risk, respectively. Therefore, the diagnostic conclusion must reflect these critical elements for accurate patient management and prognostication, as emphasized in the rigorous training at American Board of Pathology – Subspecialty in Dermatopathology University.
Incorrect
The scenario describes a patient with a history of chronic sun exposure and a new, rapidly growing lesion on the forearm. Histopathological examination reveals a nodular proliferation of atypical keratinocytes with significant nuclear pleomorphism, hyperchromasia, and keratin pearl formation. Immunohistochemical staining demonstrates strong positivity for p53 in a significant proportion of the atypical cells, with a markedly reduced or absent expression of differentiation markers like K10 in the suprabasal layers. The presence of a prominent desmoplastic stromal reaction and perineural invasion further supports a high-grade malignancy. Considering the clinical presentation and the detailed histopathological and immunohistochemical findings, the most appropriate diagnosis that aligns with the aggressive features and the observed molecular marker expression is invasive squamous cell carcinoma with high-risk features. The p53 overexpression is a common indicator of DNA damage accumulation and is frequently associated with increased aggressiveness and poor prognosis in squamous cell carcinomas. The loss of K10 suggests a profound disruption of normal keratinocyte differentiation pathways. The desmoplastic stroma and perineural invasion are well-established indicators of increased metastatic potential and local recurrence risk, respectively. Therefore, the diagnostic conclusion must reflect these critical elements for accurate patient management and prognostication, as emphasized in the rigorous training at American Board of Pathology – Subspecialty in Dermatopathology University.
-
Question 9 of 29
9. Question
A dermatopathologist at the American Board of Pathology – Subspecialty in Dermatopathology University is evaluating a skin biopsy from a patient presenting with multiple, firm, erythematous papules and nodules on the trunk. Histologically, the dermis exhibits a diffuse infiltrate of small to medium-sized lymphocytes with scant cytoplasm and hyperchromatic nuclei, predominantly located in the superficial and mid-dermis, with some perivascular accentuation. Ancillary immunohistochemical studies reveal the following: CD20 (+), CD3 (-), CD5 (+), CD43 (+), and Bcl-2 (+). Considering the differential diagnosis of cutaneous lymphoid neoplasms, which of the following immunophenotypic profiles is most consistent with the observed findings and the typical diagnostic considerations taught at the American Board of Pathology – Subspecialty in Dermatopathology University?
Correct
The question probes the understanding of the interplay between specific immunohistochemical markers and the diagnostic classification of cutaneous lymphoid neoplasms, a core competency in dermatopathology. The scenario describes a lesion with histopathological features suggestive of a B-cell lymphoma. The provided immunohistochemical panel includes CD20, CD3, CD5, CD43, and Bcl-2. In this context, CD20 positivity confirms a B-cell lineage. CD3 positivity would indicate T-cell lineage, which is not the primary consideration here given the B-cell suspicion. CD5 is a marker often found on B-cells in chronic lymphocytic leukemia/small lymphocytic lymphoma (CLL/SLL) and mantle cell lymphoma, but it can also be expressed by T-cells. CD43 is typically negative in mature B-cells but positive in some B-cell lymphomas, including CLL/SLL and some T-cell lymphomas. Bcl-2 is an anti-apoptotic protein that is often overexpressed in lymphomas, particularly follicular lymphoma and mantle cell lymphoma, and is also seen in CLL/SLL. The key to differentiating between various B-cell lymphomas lies in the combination of these markers. A B-cell lymphoma that is CD20+, CD3-, CD5+, CD43+, and Bcl-2+ strongly suggests a diagnosis of chronic lymphocytic leukemia/small lymphocytic lymphoma (CLL/SLL) or potentially a mantle cell lymphoma, depending on other markers not listed (like Cyclin D1 for mantle cell lymphoma). However, among the given options, the pattern described most closely aligns with the typical immunophenotype of CLL/SLL when it infiltrates the skin. The explanation focuses on why this specific combination of markers points towards a particular diagnostic category within cutaneous B-cell lymphomas, emphasizing the differential diagnostic considerations relevant to dermatopathology practice at the American Board of Pathology – Subspecialty in Dermatopathology University. The rationale highlights the importance of a comprehensive immunophenotypic panel for accurate classification, which is crucial for prognosis and treatment planning.
Incorrect
The question probes the understanding of the interplay between specific immunohistochemical markers and the diagnostic classification of cutaneous lymphoid neoplasms, a core competency in dermatopathology. The scenario describes a lesion with histopathological features suggestive of a B-cell lymphoma. The provided immunohistochemical panel includes CD20, CD3, CD5, CD43, and Bcl-2. In this context, CD20 positivity confirms a B-cell lineage. CD3 positivity would indicate T-cell lineage, which is not the primary consideration here given the B-cell suspicion. CD5 is a marker often found on B-cells in chronic lymphocytic leukemia/small lymphocytic lymphoma (CLL/SLL) and mantle cell lymphoma, but it can also be expressed by T-cells. CD43 is typically negative in mature B-cells but positive in some B-cell lymphomas, including CLL/SLL and some T-cell lymphomas. Bcl-2 is an anti-apoptotic protein that is often overexpressed in lymphomas, particularly follicular lymphoma and mantle cell lymphoma, and is also seen in CLL/SLL. The key to differentiating between various B-cell lymphomas lies in the combination of these markers. A B-cell lymphoma that is CD20+, CD3-, CD5+, CD43+, and Bcl-2+ strongly suggests a diagnosis of chronic lymphocytic leukemia/small lymphocytic lymphoma (CLL/SLL) or potentially a mantle cell lymphoma, depending on other markers not listed (like Cyclin D1 for mantle cell lymphoma). However, among the given options, the pattern described most closely aligns with the typical immunophenotype of CLL/SLL when it infiltrates the skin. The explanation focuses on why this specific combination of markers points towards a particular diagnostic category within cutaneous B-cell lymphomas, emphasizing the differential diagnostic considerations relevant to dermatopathology practice at the American Board of Pathology – Subspecialty in Dermatopathology University. The rationale highlights the importance of a comprehensive immunophenotypic panel for accurate classification, which is crucial for prognosis and treatment planning.
-
Question 10 of 29
10. Question
Consider a 72-year-old male with a significant history of occupational sun exposure who presents with a 2 cm exophytic nodule on the left helix of his ear. Biopsy reveals invasive squamous cell carcinoma with perineural invasion. The tumor invades into the reticular dermis. Given these findings and the educational emphasis on comprehensive patient management at American Board of Pathology – Subspecialty in Dermatopathology University, what is the most critical subsequent step in the patient’s care pathway?
Correct
The scenario describes a patient with a history of chronic sun exposure and a new, rapidly growing lesion. The histological findings of atypical keratinocytes with enlarged, pleomorphic, hyperchromatic nuclei, prominent nucleoli, dyskeratosis, and intercellular bridges, along with invasion into the dermis, are characteristic of invasive squamous cell carcinoma. The presence of perineural invasion, as noted in the description, is a critical prognostic factor that significantly increases the risk of local recurrence and regional metastasis. Therefore, the most appropriate next step in management, reflecting the rigorous standards of dermatopathology training at American Board of Pathology – Subspecialty in Dermatopathology University, is to ensure complete tumor excision with adequate margins and to consider sentinel lymph node biopsy if indicated by the depth of invasion or clinical suspicion of nodal involvement. This approach aligns with the principles of oncologic surgical pathology and the multidisciplinary care emphasized in dermatopathology. The other options, while potentially relevant in other contexts, do not directly address the immediate management of a confirmed invasive squamous cell carcinoma with concerning features like perineural invasion. Topical chemotherapy might be considered for actinic keratosis or superficial squamous cell carcinoma in situ, but not for invasive disease with perineural invasion. Observation alone is inappropriate given the invasive nature and perineural involvement. A simple shave biopsy would not be sufficient for definitive treatment of an invasive malignancy.
Incorrect
The scenario describes a patient with a history of chronic sun exposure and a new, rapidly growing lesion. The histological findings of atypical keratinocytes with enlarged, pleomorphic, hyperchromatic nuclei, prominent nucleoli, dyskeratosis, and intercellular bridges, along with invasion into the dermis, are characteristic of invasive squamous cell carcinoma. The presence of perineural invasion, as noted in the description, is a critical prognostic factor that significantly increases the risk of local recurrence and regional metastasis. Therefore, the most appropriate next step in management, reflecting the rigorous standards of dermatopathology training at American Board of Pathology – Subspecialty in Dermatopathology University, is to ensure complete tumor excision with adequate margins and to consider sentinel lymph node biopsy if indicated by the depth of invasion or clinical suspicion of nodal involvement. This approach aligns with the principles of oncologic surgical pathology and the multidisciplinary care emphasized in dermatopathology. The other options, while potentially relevant in other contexts, do not directly address the immediate management of a confirmed invasive squamous cell carcinoma with concerning features like perineural invasion. Topical chemotherapy might be considered for actinic keratosis or superficial squamous cell carcinoma in situ, but not for invasive disease with perineural invasion. Observation alone is inappropriate given the invasive nature and perineural involvement. A simple shave biopsy would not be sufficient for definitive treatment of an invasive malignancy.
-
Question 11 of 29
11. Question
Consider a challenging case presented at the American Board of Pathology – Subspecialty in Dermatopathology University, involving a 65-year-old male with multiple indurated erythematous plaques on the trunk. Histopathological examination reveals a dense lymphoid infiltrate within the dermis, predominantly perivascular and interstitial, with some nodular aggregates. Immunohistochemical stains show the infiltrate to be positive for CD20, CD79a, and BCL-6, with a subset of cells also expressing CD10. Scattered CD3-positive T-cells are present, but the majority of the infiltrate is B-cell lineage. The diagnostic question is whether this represents a primary cutaneous follicle center lymphoma or a reactive lymphoid hyperplasia. Which of the following findings, in conjunction with the existing panel, would most strongly support a diagnosis of primary cutaneous follicle center lymphoma over a reactive process or other B-cell lymphomas?
Correct
The question probes the understanding of the interplay between specific immunohistochemical markers and the diagnostic classification of cutaneous lymphoid neoplasms, particularly differentiating between primary cutaneous B-cell lymphomas and other lymphoid infiltrates. In the context of a challenging differential diagnosis, the presence of CD20, CD79a, and BCL-6 positivity, coupled with a germinal center B-cell origin, strongly supports a B-cell lymphoma. However, the crucial discriminator for a primary cutaneous follicle center lymphoma (PCFCL) versus a systemic follicular lymphoma with secondary cutaneous involvement, or even a reactive germinal center proliferation, lies in the pattern of BCL-2 expression and the overall clonal architecture. PCFCLs typically exhibit a follicular growth pattern and are often BCL-2 negative, or focally positive, reflecting their origin from germinal center B-cells that have undergone a specific maturational pathway. Conversely, a strong and diffuse BCL-2 positivity, especially in conjunction with a lack of CD10 or a discordant expression pattern, might suggest a different subtype of B-cell lymphoma or a reactive process mimicking lymphoma. The absence of T-cell markers like CD3 and CD5, and the lack of plasma cell markers such as CD138, further refine the differential. Given the scenario implies a diagnostic dilemma, the most definitive marker to confirm a primary cutaneous follicle center lymphoma, distinguishing it from other B-cell lymphomas or reactive processes, is the characteristic follicular pattern of BCL-6 expression and the typical BCL-2 negativity or focal positivity. Therefore, the absence of significant BCL-2 aberrant expression is a key feature supporting PCFCL. The calculation is conceptual: identifying the marker that most reliably distinguishes PCFCL from other possibilities based on established dermatopathology literature and the provided marker panel. The correct approach involves recognizing that while BCL-6 confirms germinal center origin, BCL-2’s pattern is critical for PCFCL diagnosis.
Incorrect
The question probes the understanding of the interplay between specific immunohistochemical markers and the diagnostic classification of cutaneous lymphoid neoplasms, particularly differentiating between primary cutaneous B-cell lymphomas and other lymphoid infiltrates. In the context of a challenging differential diagnosis, the presence of CD20, CD79a, and BCL-6 positivity, coupled with a germinal center B-cell origin, strongly supports a B-cell lymphoma. However, the crucial discriminator for a primary cutaneous follicle center lymphoma (PCFCL) versus a systemic follicular lymphoma with secondary cutaneous involvement, or even a reactive germinal center proliferation, lies in the pattern of BCL-2 expression and the overall clonal architecture. PCFCLs typically exhibit a follicular growth pattern and are often BCL-2 negative, or focally positive, reflecting their origin from germinal center B-cells that have undergone a specific maturational pathway. Conversely, a strong and diffuse BCL-2 positivity, especially in conjunction with a lack of CD10 or a discordant expression pattern, might suggest a different subtype of B-cell lymphoma or a reactive process mimicking lymphoma. The absence of T-cell markers like CD3 and CD5, and the lack of plasma cell markers such as CD138, further refine the differential. Given the scenario implies a diagnostic dilemma, the most definitive marker to confirm a primary cutaneous follicle center lymphoma, distinguishing it from other B-cell lymphomas or reactive processes, is the characteristic follicular pattern of BCL-6 expression and the typical BCL-2 negativity or focal positivity. Therefore, the absence of significant BCL-2 aberrant expression is a key feature supporting PCFCL. The calculation is conceptual: identifying the marker that most reliably distinguishes PCFCL from other possibilities based on established dermatopathology literature and the provided marker panel. The correct approach involves recognizing that while BCL-6 confirms germinal center origin, BCL-2’s pattern is critical for PCFCL diagnosis.
-
Question 12 of 29
12. Question
A 72-year-old male presents with a firm, flesh-colored nodule on the left cheek, measuring 1.5 cm. Histopathological examination reveals a poorly circumscribed invasive neoplasm composed of nests and cords of atypical basaloid cells with peripheral palisading and stromal retraction. Scattered throughout the tumor are areas of squamous differentiation and focal necrosis. Immunohistochemical stains were performed. Which of the following immunophenotypic profiles would be most indicative of a diagnosis of sebaceous carcinoma, distinguishing it from other cutaneous adnexal malignancies, in the context of a challenging differential diagnosis at the American Board of Pathology – Subspecialty in Dermatopathology University?
Correct
The question probes the understanding of immunohistochemical markers in differentiating specific types of cutaneous adnexal neoplasms, a core competency in dermatopathology. To arrive at the correct answer, one must consider the typical immunophenotypic profiles of the listed entities. Sebaceous carcinoma is characterized by a strong and diffuse expression of sebaceous differentiation markers, such as Androgen Receptor (AR) and typically exhibits positivity for EMA. Adnexal carcinoma, a broader category, can have variable markers but often lacks the specific sebaceous differentiation seen in sebaceous carcinoma. Pilomatricus carcinoma, a malignant transformation of pilomatrixoma, usually shows positivity for matrix proteins like beta-catenin and often hair follicle markers, but not the characteristic sebaceous markers. Finally, eccrine adenocarcinoma, originating from sweat glands, will typically express markers of eccrine differentiation, such as GCDFP-15 and occasionally CEA, but not sebaceous markers. Therefore, the combination of strong AR and EMA positivity, coupled with a lack of markers for other adnexal differentiation, points towards sebaceous carcinoma as the most likely diagnosis in a challenging adnexal neoplasm. This understanding is crucial for accurate diagnosis and patient management, aligning with the rigorous standards of the American Board of Pathology – Subspecialty in Dermatopathology University.
Incorrect
The question probes the understanding of immunohistochemical markers in differentiating specific types of cutaneous adnexal neoplasms, a core competency in dermatopathology. To arrive at the correct answer, one must consider the typical immunophenotypic profiles of the listed entities. Sebaceous carcinoma is characterized by a strong and diffuse expression of sebaceous differentiation markers, such as Androgen Receptor (AR) and typically exhibits positivity for EMA. Adnexal carcinoma, a broader category, can have variable markers but often lacks the specific sebaceous differentiation seen in sebaceous carcinoma. Pilomatricus carcinoma, a malignant transformation of pilomatrixoma, usually shows positivity for matrix proteins like beta-catenin and often hair follicle markers, but not the characteristic sebaceous markers. Finally, eccrine adenocarcinoma, originating from sweat glands, will typically express markers of eccrine differentiation, such as GCDFP-15 and occasionally CEA, but not sebaceous markers. Therefore, the combination of strong AR and EMA positivity, coupled with a lack of markers for other adnexal differentiation, points towards sebaceous carcinoma as the most likely diagnosis in a challenging adnexal neoplasm. This understanding is crucial for accurate diagnosis and patient management, aligning with the rigorous standards of the American Board of Pathology – Subspecialty in Dermatopathology University.
-
Question 13 of 29
13. Question
A 62-year-old male presents with a firm, flesh-colored papule on the scalp. Histopathological examination reveals a well-circumscribed dermal proliferation with nests of basaloid cells and peripheral palisading, reminiscent of a basal cell carcinoma, but with focal areas of squamous differentiation and a prominent stromal reaction. Immunohistochemical stains are performed. The tumor cells demonstrate strong positivity for cytokeratin 7 and negativity for cytokeratin 20. Considering the differential diagnosis of cutaneous neoplasms and the provided immunohistochemical findings, which lineage of adnexal neoplasm is most strongly suggested by this specific marker profile in the context of the presented morphology for a candidate at American Board of Pathology – Subspecialty in Dermatopathology University?
Correct
The question probes the understanding of the interplay between specific immunohistochemical markers and the diagnostic classification of cutaneous adnexal neoplasms, a core competency in dermatopathology. The scenario describes a lesion with features suggestive of a follicular origin, exhibiting positivity for cytokeratin 7 (CK7) and negativity for cytokeratin 20 (CK20). Cytokeratin 7 is typically expressed in epithelial cells of the pilosebaceous unit, including the outer root sheath and sebaceous glands. Cytokeratin 20, conversely, is more commonly found in urothelial and gastrointestinal epithelia, and its absence in this context helps to exclude certain differential diagnoses. The expression pattern of CK7 positivity and CK20 negativity, in conjunction with other potential markers not explicitly stated but implied by the differential diagnoses, strongly supports a diagnosis within the follicular neoplasm spectrum. Specifically, many follicular tumors, such as trichilemmomas and some types of follicular carcinomas, demonstrate this cytokeratin profile. While other adnexal tumors might show some CK7 positivity, the combination with CK20 negativity and the clinical context of a follicular neoplasm makes this the most fitting interpretation. The other options represent tumors with distinct immunohistochemical profiles. For instance, apocrine neoplasms often express GCDFP-15 and Androgen Receptor, and may show variable CK7 expression. Sebaceous neoplasms are typically positive for Androgen Receptor and oil red O stain, with variable CK7. Eccrine neoplasms, particularly ductal portions, are usually positive for CK7 and negative for CK20, but the clinical and morphological context would be crucial to differentiate from follicular tumors. Therefore, the observed marker expression most strongly aligns with a follicular lineage.
Incorrect
The question probes the understanding of the interplay between specific immunohistochemical markers and the diagnostic classification of cutaneous adnexal neoplasms, a core competency in dermatopathology. The scenario describes a lesion with features suggestive of a follicular origin, exhibiting positivity for cytokeratin 7 (CK7) and negativity for cytokeratin 20 (CK20). Cytokeratin 7 is typically expressed in epithelial cells of the pilosebaceous unit, including the outer root sheath and sebaceous glands. Cytokeratin 20, conversely, is more commonly found in urothelial and gastrointestinal epithelia, and its absence in this context helps to exclude certain differential diagnoses. The expression pattern of CK7 positivity and CK20 negativity, in conjunction with other potential markers not explicitly stated but implied by the differential diagnoses, strongly supports a diagnosis within the follicular neoplasm spectrum. Specifically, many follicular tumors, such as trichilemmomas and some types of follicular carcinomas, demonstrate this cytokeratin profile. While other adnexal tumors might show some CK7 positivity, the combination with CK20 negativity and the clinical context of a follicular neoplasm makes this the most fitting interpretation. The other options represent tumors with distinct immunohistochemical profiles. For instance, apocrine neoplasms often express GCDFP-15 and Androgen Receptor, and may show variable CK7 expression. Sebaceous neoplasms are typically positive for Androgen Receptor and oil red O stain, with variable CK7. Eccrine neoplasms, particularly ductal portions, are usually positive for CK7 and negative for CK20, but the clinical and morphological context would be crucial to differentiate from follicular tumors. Therefore, the observed marker expression most strongly aligns with a follicular lineage.
-
Question 14 of 29
14. Question
A 55-year-old male presents with a firm, flesh-colored dermal nodule on his forearm, measuring 1.2 cm. Histopathological examination reveals a proliferation of atypical spindle cells arranged in fascicles within the dermis, with some nuclear pleomorphism and occasional mitotic figures. An immunohistochemical panel is performed, showing the following results: SOX10 (strong, diffuse positive), Melan-A (focal positive), S100 (patchy positive), and SMA (negative). Considering the differential diagnosis of cutaneous spindle cell neoplasms and the importance of accurate diagnosis for patient management at the American Board of Pathology – Subspecialty in Dermatopathology University, which marker’s negative result is most crucial for definitively excluding a specific alternative diagnosis within the differential?
Correct
The question probes the understanding of the diagnostic implications of specific immunohistochemical markers in differentiating challenging cutaneous neoplasms, a core competency in dermatopathology training at the American Board of Pathology – Subspecialty in Dermatopathology University. The scenario describes a dermal nodule with atypical spindle cells. The provided immunohistochemical panel includes markers for melanocytic differentiation (SOX10, Melan-A), smooth muscle differentiation (SMA), and neural differentiation (S100). In this context, the presence of strong and diffuse positivity for SOX10 and Melan-A, coupled with focal positivity for S100, strongly suggests a melanocytic origin for the atypical spindle cells. SOX10 is a highly sensitive and specific marker for melanocytes and their neoplasms, including spindle cell nevi and desmoplastic melanoma. Melan-A is another key marker for melanocytic differentiation, typically found in epithelioid and spindle cell melanocytic lesions. S100 protein, while also a melanocytic marker, can be expressed in other neural and cartilaginous tumors, making it less specific on its own but supportive in conjunction with other markers. Conversely, smooth muscle actin (SMA) is expected to be negative in melanocytic neoplasms, and its presence would instead point towards a smooth muscle tumor like a leiomyosarcoma or a benign leiomyoma. Therefore, a negative SMA result is crucial for supporting a melanocytic diagnosis and ruling out a smooth muscle neoplasm. Considering the differential diagnoses for a spindle cell neoplasm of the skin, the described immunohistochemical profile overwhelmingly favors a melanocytic neoplasm, specifically a spindle cell nevus or potentially a desmoplastic melanoma, depending on other morphological features not detailed in the question. The question asks which marker’s *absence* is most critical for *excluding* a specific differential diagnosis. Given the positive findings for melanocytic markers, the primary differential to exclude among spindle cell tumors would be a mesenchymal neoplasm of non-melanocytic origin. Among the options provided, a smooth muscle tumor is a significant consideration for spindle cell lesions. Therefore, the *absence* of SMA positivity is critical for excluding a smooth muscle tumor, thereby reinforcing the melanocytic nature of the lesion.
Incorrect
The question probes the understanding of the diagnostic implications of specific immunohistochemical markers in differentiating challenging cutaneous neoplasms, a core competency in dermatopathology training at the American Board of Pathology – Subspecialty in Dermatopathology University. The scenario describes a dermal nodule with atypical spindle cells. The provided immunohistochemical panel includes markers for melanocytic differentiation (SOX10, Melan-A), smooth muscle differentiation (SMA), and neural differentiation (S100). In this context, the presence of strong and diffuse positivity for SOX10 and Melan-A, coupled with focal positivity for S100, strongly suggests a melanocytic origin for the atypical spindle cells. SOX10 is a highly sensitive and specific marker for melanocytes and their neoplasms, including spindle cell nevi and desmoplastic melanoma. Melan-A is another key marker for melanocytic differentiation, typically found in epithelioid and spindle cell melanocytic lesions. S100 protein, while also a melanocytic marker, can be expressed in other neural and cartilaginous tumors, making it less specific on its own but supportive in conjunction with other markers. Conversely, smooth muscle actin (SMA) is expected to be negative in melanocytic neoplasms, and its presence would instead point towards a smooth muscle tumor like a leiomyosarcoma or a benign leiomyoma. Therefore, a negative SMA result is crucial for supporting a melanocytic diagnosis and ruling out a smooth muscle neoplasm. Considering the differential diagnoses for a spindle cell neoplasm of the skin, the described immunohistochemical profile overwhelmingly favors a melanocytic neoplasm, specifically a spindle cell nevus or potentially a desmoplastic melanoma, depending on other morphological features not detailed in the question. The question asks which marker’s *absence* is most critical for *excluding* a specific differential diagnosis. Given the positive findings for melanocytic markers, the primary differential to exclude among spindle cell tumors would be a mesenchymal neoplasm of non-melanocytic origin. Among the options provided, a smooth muscle tumor is a significant consideration for spindle cell lesions. Therefore, the *absence* of SMA positivity is critical for excluding a smooth muscle tumor, thereby reinforcing the melanocytic nature of the lesion.
-
Question 15 of 29
15. Question
A 68-year-old male presents with a persistent, indurated plaque on his left forearm, exhibiting subtle ulceration. Histopathological examination reveals a dense dermal infiltrate composed of atypical cells with pleomorphic nuclei and abundant cytoplasm, admixed with reactive lymphocytes and eosinophils. Immunohistochemical stains demonstrate that the neoplastic cells are positive for CD20, CD79a, and PAX5. However, a significant subset of these atypical cells also expresses CD30. Considering the differential diagnosis for cutaneous lymphoid neoplasms at the American Board of Pathology – Subspecialty in Dermatopathology University, which of the following immunophenotypic findings would be most indicative of a specific diagnostic category given this presentation?
Correct
The question probes the understanding of the interplay between specific immunohistochemical markers and the diagnostic classification of cutaneous lymphoid neoplasms, a core competency in dermatopathology. The scenario describes a lesion with features suggestive of a B-cell lymphoma, but with atypical morphology. The key to differentiating between a reactive process and a neoplastic proliferation, particularly in challenging cases, lies in the precise interpretation of immunophenotypic profiles. In this context, the presence of CD20, CD79a, and PAX5 positivity strongly supports a B-cell lineage. However, the aberrant expression of CD30, a marker typically associated with T-cell lymphomas (like anaplastic large cell lymphoma) or Hodgkin lymphoma, in a B-cell neoplasm, points towards a specific subtype. Among the options provided, CD30 positivity in a B-cell lymphoma is a characteristic feature of **Hodgkin lymphoma, nodular lymphocyte predominant type**. While other B-cell lymphomas exist, and some may show aberrant marker expression, CD30 is a defining feature of this particular entity when seen in a B-cell context. The explanation of why other options are less likely is crucial for demonstrating nuanced understanding. For instance, while CD5 can be expressed by some B-cell lymphomas (like chronic lymphocytic leukemia/small lymphocytic lymphoma), its presence alone, without other specific markers or context, doesn’t definitively point to a specific diagnosis in this scenario, and it’s not the most salient aberrant marker in the given context. Similarly, T-cell markers like CD45RO and CD3 would be expected in T-cell lymphomas, which is contradicted by the strong B-cell lineage markers. The absence of a clear neoplastic B-cell population with typical morphology, coupled with CD30 expression, necessitates careful consideration of entities that exhibit this unusual combination. The American Board of Pathology – Subspecialty in Dermatopathology University emphasizes rigorous correlation of morphology with immunophenotype, especially in the diagnosis of cutaneous lymphomas, where subtle distinctions can have significant prognostic and therapeutic implications. Therefore, recognizing CD30 as a key aberrant marker in B-cell lymphomas is paramount.
Incorrect
The question probes the understanding of the interplay between specific immunohistochemical markers and the diagnostic classification of cutaneous lymphoid neoplasms, a core competency in dermatopathology. The scenario describes a lesion with features suggestive of a B-cell lymphoma, but with atypical morphology. The key to differentiating between a reactive process and a neoplastic proliferation, particularly in challenging cases, lies in the precise interpretation of immunophenotypic profiles. In this context, the presence of CD20, CD79a, and PAX5 positivity strongly supports a B-cell lineage. However, the aberrant expression of CD30, a marker typically associated with T-cell lymphomas (like anaplastic large cell lymphoma) or Hodgkin lymphoma, in a B-cell neoplasm, points towards a specific subtype. Among the options provided, CD30 positivity in a B-cell lymphoma is a characteristic feature of **Hodgkin lymphoma, nodular lymphocyte predominant type**. While other B-cell lymphomas exist, and some may show aberrant marker expression, CD30 is a defining feature of this particular entity when seen in a B-cell context. The explanation of why other options are less likely is crucial for demonstrating nuanced understanding. For instance, while CD5 can be expressed by some B-cell lymphomas (like chronic lymphocytic leukemia/small lymphocytic lymphoma), its presence alone, without other specific markers or context, doesn’t definitively point to a specific diagnosis in this scenario, and it’s not the most salient aberrant marker in the given context. Similarly, T-cell markers like CD45RO and CD3 would be expected in T-cell lymphomas, which is contradicted by the strong B-cell lineage markers. The absence of a clear neoplastic B-cell population with typical morphology, coupled with CD30 expression, necessitates careful consideration of entities that exhibit this unusual combination. The American Board of Pathology – Subspecialty in Dermatopathology University emphasizes rigorous correlation of morphology with immunophenotype, especially in the diagnosis of cutaneous lymphomas, where subtle distinctions can have significant prognostic and therapeutic implications. Therefore, recognizing CD30 as a key aberrant marker in B-cell lymphomas is paramount.
-
Question 16 of 29
16. Question
Consider a dermatopathology case presented to a fellow at the American Board of Pathology – Subspecialty in Dermatopathology University, involving a sun-damaged skin biopsy from the cheek of an elderly patient. The lesion exhibits subtle epidermal changes, including some lentiginous proliferation of melanocytes with mild nuclear enlargement and slight hyperchromasia, but lacks overt nesting or significant cytological atypia. The differential diagnosis includes benign lentigo simplex, lentigo maligna, and early lentigo maligna melanoma. The attending pathologist suggests using immunohistochemistry to clarify the diagnosis. Which of the following immunohistochemical markers, when considered in conjunction with the existing histological findings and clinical context, would be least helpful in definitively distinguishing between these possibilities in this specific, equivocal scenario?
Correct
The question probes the understanding of diagnostic pitfalls in differentiating benign melanocytic proliferations from early melanoma, specifically focusing on the role of immunohistochemistry in resolving ambiguous histological findings. In a scenario where a lentigo maligna melanoma is suspected but definitive features are equivocal, the pathologist would consider ancillary studies. HMB45 is a marker for melanocytic differentiation, particularly useful for identifying melanoma cells. Ki-67 is a proliferation marker, and while elevated proliferation can be seen in melanoma, it’s not specific enough to differentiate from atypical nevi. Melan-A (MART-1) is another melanocytic marker, often expressed in both benign and malignant melanocytes, but its distribution and intensity can sometimes offer clues. SOX10 is a transcription factor crucial for melanocyte development and is expressed in virtually all melanocytes, including those in nevi and melanoma. However, its utility in differentiating subtle atypia is limited compared to markers that highlight specific cellular features or aberrant expression patterns. The key to resolving equivocal lentigo maligna melanoma often lies in identifying subtle architectural disarray and cytological atypia within the epidermis and adnexal structures, which might be better appreciated with careful H&E review. When immunohistochemistry is employed, the pattern of expression and the presence of atypical melanocytes, particularly those exhibiting junctional spread with atypical morphology, are critical. A definitive diagnosis of lentigo maligna melanoma, especially in its early stages, relies on the combination of clinical context, architectural atypia (e.g., irregular nesting, single melanocyte spread), and cytological atypia (e.g., enlarged, pleomorphic nuclei, prominent nucleoli, atypical mitotic figures) within the epidermis and adnexa. While SOX10 is a valuable marker for melanocytic lineage, its broad expression across benign and malignant melanocytes makes it less discriminative for subtle atypia compared to the nuanced interpretation of H&E morphology and potentially other markers that highlight specific cellular abnormalities or proliferation patterns. Therefore, relying solely on SOX10 in a borderline case without considering the overall morphological context and other potential markers would be insufficient. The correct approach involves integrating all available data, with H&E morphology being paramount, and using immunohistochemistry judiciously to confirm or refute specific suspicions.
Incorrect
The question probes the understanding of diagnostic pitfalls in differentiating benign melanocytic proliferations from early melanoma, specifically focusing on the role of immunohistochemistry in resolving ambiguous histological findings. In a scenario where a lentigo maligna melanoma is suspected but definitive features are equivocal, the pathologist would consider ancillary studies. HMB45 is a marker for melanocytic differentiation, particularly useful for identifying melanoma cells. Ki-67 is a proliferation marker, and while elevated proliferation can be seen in melanoma, it’s not specific enough to differentiate from atypical nevi. Melan-A (MART-1) is another melanocytic marker, often expressed in both benign and malignant melanocytes, but its distribution and intensity can sometimes offer clues. SOX10 is a transcription factor crucial for melanocyte development and is expressed in virtually all melanocytes, including those in nevi and melanoma. However, its utility in differentiating subtle atypia is limited compared to markers that highlight specific cellular features or aberrant expression patterns. The key to resolving equivocal lentigo maligna melanoma often lies in identifying subtle architectural disarray and cytological atypia within the epidermis and adnexal structures, which might be better appreciated with careful H&E review. When immunohistochemistry is employed, the pattern of expression and the presence of atypical melanocytes, particularly those exhibiting junctional spread with atypical morphology, are critical. A definitive diagnosis of lentigo maligna melanoma, especially in its early stages, relies on the combination of clinical context, architectural atypia (e.g., irregular nesting, single melanocyte spread), and cytological atypia (e.g., enlarged, pleomorphic nuclei, prominent nucleoli, atypical mitotic figures) within the epidermis and adnexa. While SOX10 is a valuable marker for melanocytic lineage, its broad expression across benign and malignant melanocytes makes it less discriminative for subtle atypia compared to the nuanced interpretation of H&E morphology and potentially other markers that highlight specific cellular abnormalities or proliferation patterns. Therefore, relying solely on SOX10 in a borderline case without considering the overall morphological context and other potential markers would be insufficient. The correct approach involves integrating all available data, with H&E morphology being paramount, and using immunohistochemistry judiciously to confirm or refute specific suspicions.
-
Question 17 of 29
17. Question
Consider a 72-year-old gentleman presenting with a firm, flesh-colored nodule on the dorsal aspect of his left hand. Histopathological examination reveals a dermal proliferation of atypical spindle cells arranged in fascicles and storiform patterns, with focal pleomorphism and mitotic activity. The differential diagnosis includes melanoma with spindle cell morphology, atypical fibroxanthoma, and cutaneous leiomyosarcoma. Which immunohistochemical panel would be most crucial for definitively establishing a melanocytic lineage and differentiating it from other spindle cell neoplasms in the context of dermatopathology training at American Board of Pathology – Subspecialty in Dermatopathology University?
Correct
The question probes the understanding of the interplay between specific immunohistochemical markers and the diagnostic classification of cutaneous neoplasms, particularly in differentiating challenging cases that mimic other entities. The scenario describes a lesion with features suggestive of a spindle cell neoplasm. The differential diagnosis would include entities like atypical fibroxanthoma, leiomyosarcoma, malignant peripheral nerve sheath tumor, and melanoma with spindle cell morphology. To arrive at the correct answer, one must consider the typical immunophenotypic profiles of these differential diagnoses. * **Melanoma with spindle cell morphology:** Typically expresses S100 protein and SOX10, and often Melan-A (MART-1) and Tyrosinase. * **Atypical fibroxanthoma:** Often expresses CD10, CD68, and vimentin. It is typically negative for melanocytic markers, cytokeratins, and smooth muscle actin. * **Leiomyosarcoma:** Expresses smooth muscle actin (SMA), desmin, and caldesmon. It is usually negative for melanocytic markers and cytokeratins. * **Malignant peripheral nerve sheath tumor:** Expresses SOX10, S100 protein (though often focally or weakly), and vimentin. It may express EMA. Given the provided scenario of a nodular dermal lesion with atypical spindle cells and the need to distinguish it from melanoma, the most informative panel would include markers that definitively differentiate melanocytic differentiation from other spindle cell origins. S100 protein is a sensitive marker for melanoma, and while SOX10 is also highly sensitive and specific for melanocytic lineage, its expression can be seen in other neural crest-derived tumors. Melan-A (MART-1) is a highly specific marker for melanocytic differentiation and is crucial for confirming the melanocytic origin of spindle cell neoplasms, especially when S100 or SOX10 expression might be equivocal or present in other differential diagnoses. Therefore, a panel including S100, SOX10, and Melan-A is essential for accurate diagnosis and differentiation from non-melanocytic spindle cell tumors. The correct approach involves selecting the panel that provides the most robust differentiation between melanocytic and non-melanocytic spindle cell proliferations. A combination of S100, SOX10, and Melan-A offers high sensitivity and specificity for identifying melanocytic differentiation, which is critical in distinguishing melanoma from other spindle cell neoplasms that might share some morphological features. This comprehensive panel directly addresses the diagnostic challenge presented in the question, ensuring a definitive diagnosis for the American Board of Pathology – Subspecialty in Dermatopathology University curriculum’s emphasis on nuanced diagnostic interpretation.
Incorrect
The question probes the understanding of the interplay between specific immunohistochemical markers and the diagnostic classification of cutaneous neoplasms, particularly in differentiating challenging cases that mimic other entities. The scenario describes a lesion with features suggestive of a spindle cell neoplasm. The differential diagnosis would include entities like atypical fibroxanthoma, leiomyosarcoma, malignant peripheral nerve sheath tumor, and melanoma with spindle cell morphology. To arrive at the correct answer, one must consider the typical immunophenotypic profiles of these differential diagnoses. * **Melanoma with spindle cell morphology:** Typically expresses S100 protein and SOX10, and often Melan-A (MART-1) and Tyrosinase. * **Atypical fibroxanthoma:** Often expresses CD10, CD68, and vimentin. It is typically negative for melanocytic markers, cytokeratins, and smooth muscle actin. * **Leiomyosarcoma:** Expresses smooth muscle actin (SMA), desmin, and caldesmon. It is usually negative for melanocytic markers and cytokeratins. * **Malignant peripheral nerve sheath tumor:** Expresses SOX10, S100 protein (though often focally or weakly), and vimentin. It may express EMA. Given the provided scenario of a nodular dermal lesion with atypical spindle cells and the need to distinguish it from melanoma, the most informative panel would include markers that definitively differentiate melanocytic differentiation from other spindle cell origins. S100 protein is a sensitive marker for melanoma, and while SOX10 is also highly sensitive and specific for melanocytic lineage, its expression can be seen in other neural crest-derived tumors. Melan-A (MART-1) is a highly specific marker for melanocytic differentiation and is crucial for confirming the melanocytic origin of spindle cell neoplasms, especially when S100 or SOX10 expression might be equivocal or present in other differential diagnoses. Therefore, a panel including S100, SOX10, and Melan-A is essential for accurate diagnosis and differentiation from non-melanocytic spindle cell tumors. The correct approach involves selecting the panel that provides the most robust differentiation between melanocytic and non-melanocytic spindle cell proliferations. A combination of S100, SOX10, and Melan-A offers high sensitivity and specificity for identifying melanocytic differentiation, which is critical in distinguishing melanoma from other spindle cell neoplasms that might share some morphological features. This comprehensive panel directly addresses the diagnostic challenge presented in the question, ensuring a definitive diagnosis for the American Board of Pathology – Subspecialty in Dermatopathology University curriculum’s emphasis on nuanced diagnostic interpretation.
-
Question 18 of 29
18. Question
A 45-year-old individual presents with a pigmented macule on the dorsal aspect of the forearm, measuring 0.7 cm in diameter. Excision biopsy reveals a proliferation of melanocytes predominantly at the dermal-epidermal junction, with some nests exhibiting irregular shapes and varying sizes. Critically, scattered single atypical melanocytes are noted extending into the papillary dermis, displaying enlarged, pleomorphic nuclei with prominent nucleoli and hyperchromasia. A mild to moderate lymphocytic infiltrate is present within the superficial dermis. Considering the histological findings and the need for precise diagnostic categorization within the context of dermatopathology training at American Board of Pathology – Subspecialty in Dermatopathology University, which of the following interpretations most accurately reflects the observed pathology and guides subsequent management?
Correct
The question probes the understanding of diagnostic criteria for a specific melanocytic proliferation, requiring differentiation between benign and potentially malignant entities based on subtle histological features. The scenario describes a lesion with architectural disorder, atypical melanocytes at the dermal-epidermal junction and within the papillary dermis, and a lymphocytic infiltrate. Key features to consider for a diagnosis of melanoma in situ or early invasive melanoma include significant cytological atypia (enlarged, pleomorphic nuclei, prominent nucleoli, hyperchromasia), presence of single atypical melanocytes at the dermal-epidermal junction, and a significant number of atypical melanocytes extending into the dermis. The description of “significant cytological atypia” and “melanocytes extending into the papillary dermis” points towards a diagnosis beyond a simple benign nevus. While lentiginous proliferation and mild atypia can be seen in some benign nevi, the combination of architectural disorder, significant cytological atypia, and dermal extension strongly suggests a malignant or premalignant process. Specifically, the presence of atypical melanocytes within the papillary dermis, even if not overtly invasive, raises concern for melanoma. The lymphocytic infiltrate is a common host response to atypical melanocytes. Therefore, the most appropriate interpretation, considering the need for further evaluation and potential for malignancy, is a diagnosis that reflects this concern. The provided options are designed to test the ability to discern the degree of atypia and the significance of dermal involvement. A benign nevus would typically lack significant cytological atypia and dermal extension of atypical melanocytes. Dysplastic nevi can show some atypia and architectural disarray, but the described features, particularly the dermal component of atypical melanocytes, push the diagnosis towards melanoma. A Spitz nevus, while showing some atypia, typically has a more symmetrical architecture and often lacks the significant cytological atypia described. The correct interpretation necessitates recognizing the potential for melanoma based on the described histological findings, guiding appropriate management and further investigation.
Incorrect
The question probes the understanding of diagnostic criteria for a specific melanocytic proliferation, requiring differentiation between benign and potentially malignant entities based on subtle histological features. The scenario describes a lesion with architectural disorder, atypical melanocytes at the dermal-epidermal junction and within the papillary dermis, and a lymphocytic infiltrate. Key features to consider for a diagnosis of melanoma in situ or early invasive melanoma include significant cytological atypia (enlarged, pleomorphic nuclei, prominent nucleoli, hyperchromasia), presence of single atypical melanocytes at the dermal-epidermal junction, and a significant number of atypical melanocytes extending into the dermis. The description of “significant cytological atypia” and “melanocytes extending into the papillary dermis” points towards a diagnosis beyond a simple benign nevus. While lentiginous proliferation and mild atypia can be seen in some benign nevi, the combination of architectural disorder, significant cytological atypia, and dermal extension strongly suggests a malignant or premalignant process. Specifically, the presence of atypical melanocytes within the papillary dermis, even if not overtly invasive, raises concern for melanoma. The lymphocytic infiltrate is a common host response to atypical melanocytes. Therefore, the most appropriate interpretation, considering the need for further evaluation and potential for malignancy, is a diagnosis that reflects this concern. The provided options are designed to test the ability to discern the degree of atypia and the significance of dermal involvement. A benign nevus would typically lack significant cytological atypia and dermal extension of atypical melanocytes. Dysplastic nevi can show some atypia and architectural disarray, but the described features, particularly the dermal component of atypical melanocytes, push the diagnosis towards melanoma. A Spitz nevus, while showing some atypia, typically has a more symmetrical architecture and often lacks the significant cytological atypia described. The correct interpretation necessitates recognizing the potential for melanoma based on the described histological findings, guiding appropriate management and further investigation.
-
Question 19 of 29
19. Question
A dermatopathology fellow at the American Board of Pathology – Subspecialty in Dermatopathology University is evaluating a biopsy from the dorsal aspect of the hand of a 65-year-old male, revealing a dermal proliferation of spindle-shaped cells with moderate nuclear pleomorphism and focal mitotic activity. Immunohistochemical stains demonstrate strong and diffuse positivity for SOX10 and focal positivity for S100, but complete negativity for HMB45 and Melan-A. Considering the differential diagnosis of cutaneous spindle cell neoplasms, which of the following interpretations best aligns with the observed histopathological and immunohistochemical findings in the context of advanced dermatopathology diagnostics?
Correct
The question probes the understanding of the interplay between specific immunohistochemical markers and the diagnostic classification of cutaneous neoplasms, particularly in differentiating challenging cases that might arise in a dermatopathology fellowship at the American Board of Pathology – Subspecialty in Dermatopathology University. The scenario describes a biopsy with features suggestive of a melanocytic lesion, but with atypical architectural and cytological elements that necessitate precise marker utilization. Consider a case where a biopsy shows a dermal proliferation of spindle cells with scant cytoplasm and pleomorphic nuclei, arranged in fascicles. Immunohistochemical stains reveal positivity for SOX10 and S100, but negativity for HMB45 and Melan-A. This pattern of reactivity is crucial for distinguishing between various melanocytic proliferations and other spindle cell tumors. SOX10 and S100 are broadly expressed in melanocytes and their precursors, making them sensitive markers for melanocytic differentiation. However, their presence alone does not confirm a benign nevus or a malignant melanoma. The negativity for HMB45 and Melan-A, which are typically expressed in mature melanocytes and melanoma, is a key differentiator. In the context of distinguishing a benign spindle cell nevus from a melanoma or a desmoplastic melanoma, the absence of HMB45 and Melan-A in the dermal component, especially when combined with positive SOX10 and S100, strongly suggests a desmoplastic melanoma or a Spitzoid lesion with desmoplastic features. However, desmoplastic melanoma often shows variable S100 and SOX10 expression and can be negative for HMB45/Melan-A in the desmoplastic component. A crucial point for advanced trainees is recognizing that while HMB45 and Melan-A are important for melanoma diagnosis, their absence in a S100/SOX10 positive spindle cell lesion does not automatically exclude melanoma, particularly desmoplastic melanoma. Instead, the overall pattern, including nuclear atypia, mitotic activity, and stromal desmoplasia, alongside the precise marker profile, guides the diagnosis. The correct approach involves recognizing that while HMB45 and Melan-A are often positive in melanoma, their absence in a S100/SOX10 positive spindle cell neoplasm, particularly in the context of significant stromal desmoplasia and cellular atypia, points towards a desmoplastic melanoma. This distinction is critical for patient management and prognosis, aligning with the rigorous diagnostic standards expected at the American Board of Pathology – Subspecialty in Dermatopathology University. The differential diagnosis would also include other spindle cell tumors, but the melanocytic markers are paramount here. Therefore, the combination of positive SOX10 and S100 with negative HMB45 and Melan-A in a desmoplastic spindle cell neoplasm is most consistent with desmoplastic melanoma.
Incorrect
The question probes the understanding of the interplay between specific immunohistochemical markers and the diagnostic classification of cutaneous neoplasms, particularly in differentiating challenging cases that might arise in a dermatopathology fellowship at the American Board of Pathology – Subspecialty in Dermatopathology University. The scenario describes a biopsy with features suggestive of a melanocytic lesion, but with atypical architectural and cytological elements that necessitate precise marker utilization. Consider a case where a biopsy shows a dermal proliferation of spindle cells with scant cytoplasm and pleomorphic nuclei, arranged in fascicles. Immunohistochemical stains reveal positivity for SOX10 and S100, but negativity for HMB45 and Melan-A. This pattern of reactivity is crucial for distinguishing between various melanocytic proliferations and other spindle cell tumors. SOX10 and S100 are broadly expressed in melanocytes and their precursors, making them sensitive markers for melanocytic differentiation. However, their presence alone does not confirm a benign nevus or a malignant melanoma. The negativity for HMB45 and Melan-A, which are typically expressed in mature melanocytes and melanoma, is a key differentiator. In the context of distinguishing a benign spindle cell nevus from a melanoma or a desmoplastic melanoma, the absence of HMB45 and Melan-A in the dermal component, especially when combined with positive SOX10 and S100, strongly suggests a desmoplastic melanoma or a Spitzoid lesion with desmoplastic features. However, desmoplastic melanoma often shows variable S100 and SOX10 expression and can be negative for HMB45/Melan-A in the desmoplastic component. A crucial point for advanced trainees is recognizing that while HMB45 and Melan-A are important for melanoma diagnosis, their absence in a S100/SOX10 positive spindle cell lesion does not automatically exclude melanoma, particularly desmoplastic melanoma. Instead, the overall pattern, including nuclear atypia, mitotic activity, and stromal desmoplasia, alongside the precise marker profile, guides the diagnosis. The correct approach involves recognizing that while HMB45 and Melan-A are often positive in melanoma, their absence in a S100/SOX10 positive spindle cell neoplasm, particularly in the context of significant stromal desmoplasia and cellular atypia, points towards a desmoplastic melanoma. This distinction is critical for patient management and prognosis, aligning with the rigorous diagnostic standards expected at the American Board of Pathology – Subspecialty in Dermatopathology University. The differential diagnosis would also include other spindle cell tumors, but the melanocytic markers are paramount here. Therefore, the combination of positive SOX10 and S100 with negative HMB45 and Melan-A in a desmoplastic spindle cell neoplasm is most consistent with desmoplastic melanoma.
-
Question 20 of 29
20. Question
A 68-year-old male presents with a slowly growing, erythematous, slightly indurated plaque on his left forearm, present for approximately 18 months. Histopathological examination reveals a diffuse infiltrate of small to medium-sized lymphoid cells within the dermis, with some nodular aggregates. The cells exhibit mild nuclear pleomorphism and scant cytoplasm. Immunohistochemical stains are performed. The neoplastic cells are positive for CD20, CD5, and BCL2, but negative for CD3 and CD43. Ki-67 proliferation index is approximately 15%. Considering the differential diagnosis of a cutaneous B-cell lymphoma, which of the following immunohistochemical profiles would most strongly support a diagnosis of primary cutaneous follicular lymphoma over primary cutaneous marginal zone lymphoma?
Correct
The question probes the understanding of the interplay between specific immunohistochemical markers and the diagnostic classification of cutaneous lymphoid neoplasms, a core competency in dermatopathology. The scenario describes a lesion with features suggestive of a primary cutaneous B-cell lymphoma, but with atypical morphology. The key to differentiating between a follicular lymphoma (FL) and a marginal zone lymphoma (MZL) often lies in the expression patterns of CD10 and BCL6. Follicular lymphoma typically exhibits a CD10-positive and BCL6-positive phenotype, reflecting its germinal center origin. Marginal zone lymphoma, on the other hand, is generally CD10-negative and BCL6-negative, arising from the marginal zone B cells. While CD20 is a pan-B-cell marker and would be expected in both, and BCL2 can be positive in both FL and some MZL, the definitive distinction in this context, especially with the described morphology, hinges on the CD10 and BCL6 expression. Therefore, a CD10-positive, BCL6-positive profile strongly favors follicular lymphoma over marginal zone lymphoma. The explanation emphasizes that accurate interpretation of these markers, in conjunction with morphology and clinical context, is paramount for precise diagnosis and patient management, aligning with the rigorous standards of dermatopathology training at the American Board of Pathology – Subspecialty in Dermatopathology University.
Incorrect
The question probes the understanding of the interplay between specific immunohistochemical markers and the diagnostic classification of cutaneous lymphoid neoplasms, a core competency in dermatopathology. The scenario describes a lesion with features suggestive of a primary cutaneous B-cell lymphoma, but with atypical morphology. The key to differentiating between a follicular lymphoma (FL) and a marginal zone lymphoma (MZL) often lies in the expression patterns of CD10 and BCL6. Follicular lymphoma typically exhibits a CD10-positive and BCL6-positive phenotype, reflecting its germinal center origin. Marginal zone lymphoma, on the other hand, is generally CD10-negative and BCL6-negative, arising from the marginal zone B cells. While CD20 is a pan-B-cell marker and would be expected in both, and BCL2 can be positive in both FL and some MZL, the definitive distinction in this context, especially with the described morphology, hinges on the CD10 and BCL6 expression. Therefore, a CD10-positive, BCL6-positive profile strongly favors follicular lymphoma over marginal zone lymphoma. The explanation emphasizes that accurate interpretation of these markers, in conjunction with morphology and clinical context, is paramount for precise diagnosis and patient management, aligning with the rigorous standards of dermatopathology training at the American Board of Pathology – Subspecialty in Dermatopathology University.
-
Question 21 of 29
21. Question
Consider a biopsy specimen from a 55-year-old male presenting with a persistent erythematous, slightly scaly plaque on his left forearm, a common presentation encountered in dermatopathology practice at American Board of Pathology – Subspecialty in Dermatopathology University. Histopathological examination reveals a superficial and deep perivascular and band-like dermal infiltrate composed of atypical lymphocytes. Immunohistochemical studies demonstrate that the infiltrate is positive for CD3, CD4, and TIA-1, while negative for CD8 and CD30. Based on these findings, which of the following diagnoses is most strongly supported?
Correct
The question probes the understanding of the interplay between specific immunohistochemical markers and the diagnostic classification of cutaneous lymphoid neoplasms, a core competency in dermatopathology. The scenario describes a lesion with features suggestive of a T-cell lymphoma. The key diagnostic markers provided are CD3, CD4, CD8, CD30, and TIA-1. A CD3-positive, CD4-positive, CD8-negative, CD30-negative, and TIA-1-positive infiltrate, particularly in the context of a superficial perivascular and band-like dermal infiltrate with epidermal spongiosis and interface changes, strongly points towards a diagnosis of Mycosis Fungoides (MF). CD3 confirms T-cell origin. CD4 positivity is typical for the helper T-cell phenotype commonly seen in MF. The absence of CD8 further supports a CD4+ T-cell process. Crucially, CD30 negativity helps differentiate MF from Hodgkin lymphoma or anaplastic large cell lymphoma, which often exhibit CD30 positivity. TIA-1 (T-cell intracellular antigen 1) is a cytoplasmic marker often expressed in cytotoxic T-cells and NK cells, and its presence in a neoplastic T-cell population can be seen in various T-cell lymphomas, including MF, but its positivity in conjunction with the other markers solidifies the MF diagnosis over other possibilities. For instance, a CD4+ cutaneous T-cell lymphoma that is CD30-positive would lean towards a different classification. A B-cell lymphoma would be characterized by B-cell markers such as CD20 and CD79a, and would be negative for T-cell markers. While other CD4+ T-cell lymphomas exist, the combination of findings, especially the absence of CD30 and the typical pattern, makes Mycosis Fungoides the most fitting diagnosis among the provided options. The explanation emphasizes the rationale behind each marker’s contribution to the differential diagnosis, highlighting how their specific expression patterns guide the pathologist towards the correct classification, a critical skill for advanced dermatopathology trainees at American Board of Pathology – Subspecialty in Dermatopathology University.
Incorrect
The question probes the understanding of the interplay between specific immunohistochemical markers and the diagnostic classification of cutaneous lymphoid neoplasms, a core competency in dermatopathology. The scenario describes a lesion with features suggestive of a T-cell lymphoma. The key diagnostic markers provided are CD3, CD4, CD8, CD30, and TIA-1. A CD3-positive, CD4-positive, CD8-negative, CD30-negative, and TIA-1-positive infiltrate, particularly in the context of a superficial perivascular and band-like dermal infiltrate with epidermal spongiosis and interface changes, strongly points towards a diagnosis of Mycosis Fungoides (MF). CD3 confirms T-cell origin. CD4 positivity is typical for the helper T-cell phenotype commonly seen in MF. The absence of CD8 further supports a CD4+ T-cell process. Crucially, CD30 negativity helps differentiate MF from Hodgkin lymphoma or anaplastic large cell lymphoma, which often exhibit CD30 positivity. TIA-1 (T-cell intracellular antigen 1) is a cytoplasmic marker often expressed in cytotoxic T-cells and NK cells, and its presence in a neoplastic T-cell population can be seen in various T-cell lymphomas, including MF, but its positivity in conjunction with the other markers solidifies the MF diagnosis over other possibilities. For instance, a CD4+ cutaneous T-cell lymphoma that is CD30-positive would lean towards a different classification. A B-cell lymphoma would be characterized by B-cell markers such as CD20 and CD79a, and would be negative for T-cell markers. While other CD4+ T-cell lymphomas exist, the combination of findings, especially the absence of CD30 and the typical pattern, makes Mycosis Fungoides the most fitting diagnosis among the provided options. The explanation emphasizes the rationale behind each marker’s contribution to the differential diagnosis, highlighting how their specific expression patterns guide the pathologist towards the correct classification, a critical skill for advanced dermatopathology trainees at American Board of Pathology – Subspecialty in Dermatopathology University.
-
Question 22 of 29
22. Question
Consider a challenging case presented to the American Board of Pathology – Subspecialty in Dermatopathology University, involving a patient with persistent, erythematous, scaly plaques. Histopathological examination reveals a superficial perivascular and band-like lymphocytic infiltrate with some focal epidermotropism. The differential diagnosis includes mycosis fungoides, benign reactive lymphoid hyperplasia, and cutaneous melanoma with a significant inflammatory infiltrate. Which immunohistochemical panel, when interpreted in conjunction with the clinical presentation and H&E findings, would be most instrumental in definitively diagnosing mycosis fungoides in this scenario?
Correct
The question probes the understanding of immunohistochemical markers in differentiating challenging cutaneous neoplasms, specifically focusing on the distinction between primary cutaneous lymphomas and other entities that can mimic them. In the context of a difficult differential diagnosis, such as distinguishing a mycosis fungoides (MF) from a reactive lymphoid hyperplasia or a melanoma with lymphoid infiltration, careful selection of markers is paramount. For MF, a CD4+ helper T-cell lymphoma, key markers include CD3, CD4, and CD45RO. Crucially, the aberrant expression of CD45RO (a marker of memory T cells) and the presence of epidermotropism (T cells within the epidermis) are characteristic. While CD3 and CD4 are generally positive in MF, their presence alone is not sufficient for diagnosis. CD8 is typically negative or weakly expressed in the neoplastic T cells of MF. CD20 would be positive in B-cell lymphomas, which are distinct from T-cell lymphomas like MF. S100 is a marker for melanocytes and melanoma, and while it can be seen in some reactive stromal cells, it is not a primary marker for distinguishing MF from other lymphoid proliferations. Therefore, a panel that includes CD3, CD4, CD45RO, and CD8, along with assessment for epidermotropism, is essential. The absence of CD20 and S100 would further support a T-cell lineage and rule out B-cell lymphoma or melanoma, respectively. The combination of CD4 positivity, CD45RO positivity, and CD8 negativity in the context of epidermotropism is highly suggestive of mycosis fungoides, aligning with established diagnostic criteria used in dermatopathology practice at institutions like American Board of Pathology – Subspecialty in Dermatopathology University.
Incorrect
The question probes the understanding of immunohistochemical markers in differentiating challenging cutaneous neoplasms, specifically focusing on the distinction between primary cutaneous lymphomas and other entities that can mimic them. In the context of a difficult differential diagnosis, such as distinguishing a mycosis fungoides (MF) from a reactive lymphoid hyperplasia or a melanoma with lymphoid infiltration, careful selection of markers is paramount. For MF, a CD4+ helper T-cell lymphoma, key markers include CD3, CD4, and CD45RO. Crucially, the aberrant expression of CD45RO (a marker of memory T cells) and the presence of epidermotropism (T cells within the epidermis) are characteristic. While CD3 and CD4 are generally positive in MF, their presence alone is not sufficient for diagnosis. CD8 is typically negative or weakly expressed in the neoplastic T cells of MF. CD20 would be positive in B-cell lymphomas, which are distinct from T-cell lymphomas like MF. S100 is a marker for melanocytes and melanoma, and while it can be seen in some reactive stromal cells, it is not a primary marker for distinguishing MF from other lymphoid proliferations. Therefore, a panel that includes CD3, CD4, CD45RO, and CD8, along with assessment for epidermotropism, is essential. The absence of CD20 and S100 would further support a T-cell lineage and rule out B-cell lymphoma or melanoma, respectively. The combination of CD4 positivity, CD45RO positivity, and CD8 negativity in the context of epidermotropism is highly suggestive of mycosis fungoides, aligning with established diagnostic criteria used in dermatopathology practice at institutions like American Board of Pathology – Subspecialty in Dermatopathology University.
-
Question 23 of 29
23. Question
A 45-year-old individual presents with a pigmented macule on the forearm exhibiting subtle architectural atypia and mild cytological variations on initial biopsy. The pathologist is considering a differential diagnosis that includes a benign atypical nevus versus early-stage melanoma. To definitively stratify this lesion and guide further patient management, which of the following immunohistochemical markers, when assessed in conjunction with standard H&E morphology and clinical correlation, would offer the most discriminative value in distinguishing between these two entities for the American Board of Pathology – Subspecialty in Dermatopathology University curriculum?
Correct
The question probes the understanding of immunohistochemical marker utility in differentiating challenging melanocytic proliferations, a core competency in dermatopathology. The scenario describes a lesion with features suggestive of both a benign nevus and early melanoma, necessitating precise diagnostic tools. While H&E staining is the cornerstone, ancillary immunohistochemistry plays a critical role in resolving diagnostic ambiguity. Ki-67 is a proliferation marker, and while elevated levels can be seen in melanoma, it is not specific enough for definitive differentiation in this context, as some atypical nevi can also show increased Ki-67. SOX10 is a pan-melanocytic marker, crucial for identifying melanocytes and their lineage, and would be positive in both nevi and melanoma, thus not discriminatory between them. Melan-A (MART-1) is a melanocytic differentiation antigen, also expressed in both benign and malignant melanocytes, making it useful for confirming melanocytic origin but not for distinguishing between them. MITF (Microphthalmia-associated transcription factor) is a master regulator of melanocyte development and survival. In the context of melanocytic lesions, its expression pattern can be informative. While typically expressed in both nevi and melanomas, subtle differences in nuclear staining intensity and distribution, particularly when combined with other markers, can aid in assessment. However, the most critical marker for distinguishing atypical melanocytic proliferations, especially in the context of potential melanoma, is the assessment of proliferation and atypicality. PRAME (Preferentially expressed antigen in melanoma) is a recently validated marker that shows significantly higher and more widespread expression in melanoma compared to benign melanocytic nevi, even in early stages. Its differential expression is a key factor in resolving diagnostic dilemmas where conventional morphology is equivocal. Therefore, a panel including PRAME, along with markers like SOX10 and Melan-A for lineage confirmation and potentially Ki-67 for proliferation assessment, would be the most effective approach. Among the options provided, focusing on PRAME as a key differentiator is the most accurate.
Incorrect
The question probes the understanding of immunohistochemical marker utility in differentiating challenging melanocytic proliferations, a core competency in dermatopathology. The scenario describes a lesion with features suggestive of both a benign nevus and early melanoma, necessitating precise diagnostic tools. While H&E staining is the cornerstone, ancillary immunohistochemistry plays a critical role in resolving diagnostic ambiguity. Ki-67 is a proliferation marker, and while elevated levels can be seen in melanoma, it is not specific enough for definitive differentiation in this context, as some atypical nevi can also show increased Ki-67. SOX10 is a pan-melanocytic marker, crucial for identifying melanocytes and their lineage, and would be positive in both nevi and melanoma, thus not discriminatory between them. Melan-A (MART-1) is a melanocytic differentiation antigen, also expressed in both benign and malignant melanocytes, making it useful for confirming melanocytic origin but not for distinguishing between them. MITF (Microphthalmia-associated transcription factor) is a master regulator of melanocyte development and survival. In the context of melanocytic lesions, its expression pattern can be informative. While typically expressed in both nevi and melanomas, subtle differences in nuclear staining intensity and distribution, particularly when combined with other markers, can aid in assessment. However, the most critical marker for distinguishing atypical melanocytic proliferations, especially in the context of potential melanoma, is the assessment of proliferation and atypicality. PRAME (Preferentially expressed antigen in melanoma) is a recently validated marker that shows significantly higher and more widespread expression in melanoma compared to benign melanocytic nevi, even in early stages. Its differential expression is a key factor in resolving diagnostic dilemmas where conventional morphology is equivocal. Therefore, a panel including PRAME, along with markers like SOX10 and Melan-A for lineage confirmation and potentially Ki-67 for proliferation assessment, would be the most effective approach. Among the options provided, focusing on PRAME as a key differentiator is the most accurate.
-
Question 24 of 29
24. Question
A 68-year-old individual presents with a slowly growing, asymptomatic papule on the left forearm. Histopathological examination reveals a dense infiltrate within the dermis, predominantly composed of small to medium-sized lymphoid cells. Within this infiltrate, there are numerous reactive germinal centers surrounded by a mantle of smaller lymphocytes. Scattered larger lymphoid cells and eosinophils are also noted. Immunohistochemical staining is performed. Considering the differential diagnoses for such a lesion, which of the following immunohistochemical markers would be most crucial in definitively establishing a diagnosis of follicular lymphoma over other B-cell lymphomas that might present with similar morphological features at the American Board of Pathology – Subspecialty in Dermatopathology University?
Correct
The question assesses the understanding of the interplay between specific immunohistochemical markers and the diagnostic interpretation of cutaneous lymphoid proliferations, a core competency in dermatopathology. The scenario describes a lesion with features suggestive of a cutaneous B-cell lymphoma, specifically a follicular lymphoma or a marginal zone lymphoma, given the presence of germinal centers and a mixed infiltrate. The key to differentiating these entities, and indeed many lymphoid neoplasms, lies in assessing the clonality of the B-cells and identifying specific lineage markers. For follicular lymphoma, a neoplastic proliferation of germinal center B-cells, the characteristic translocation is t(14;18), which results in the overexpression of BCL2. Therefore, BCL2 positivity in the neoplastic follicles, typically with a follicular pattern of staining, is a crucial diagnostic marker. CD10 is also often positive in germinal center B-cells, supporting this lineage. CD20 highlights the B-cell population, and CD3 highlights the T-cell infiltrate, which is expected in reactive germinal centers. However, BCL2 is the most specific marker for follicular lymphoma in this context. Marginal zone lymphomas, while also B-cell lymphomas, often arise from the marginal zone of lymphoid follicles or extranodal lymphoid tissue. They can exhibit varying immunophenotypes, but BCL2 positivity is less consistently observed in the neoplastic cells compared to follicular lymphoma. CD5 is typically negative in both follicular and marginal zone lymphomas, but can be seen in mantle cell lymphoma, which is another differential. CD30 is generally negative in low-grade lymphomas like follicular and marginal zone lymphomas, but can be positive in anaplastic large cell lymphoma or Hodgkin lymphoma, which are less likely given the described morphology. Therefore, the presence of BCL2 positivity in a significant proportion of the lymphoid cells, particularly within the follicular structures, strongly supports a diagnosis of follicular lymphoma and is the most critical ancillary finding among the options provided for differentiating it from other B-cell lymphomas that might present with similar morphology. The explanation focuses on the rationale behind selecting BCL2 as the most discriminative marker in this specific diagnostic context, emphasizing its role in identifying the neoplastic B-cell population characteristic of follicular lymphoma.
Incorrect
The question assesses the understanding of the interplay between specific immunohistochemical markers and the diagnostic interpretation of cutaneous lymphoid proliferations, a core competency in dermatopathology. The scenario describes a lesion with features suggestive of a cutaneous B-cell lymphoma, specifically a follicular lymphoma or a marginal zone lymphoma, given the presence of germinal centers and a mixed infiltrate. The key to differentiating these entities, and indeed many lymphoid neoplasms, lies in assessing the clonality of the B-cells and identifying specific lineage markers. For follicular lymphoma, a neoplastic proliferation of germinal center B-cells, the characteristic translocation is t(14;18), which results in the overexpression of BCL2. Therefore, BCL2 positivity in the neoplastic follicles, typically with a follicular pattern of staining, is a crucial diagnostic marker. CD10 is also often positive in germinal center B-cells, supporting this lineage. CD20 highlights the B-cell population, and CD3 highlights the T-cell infiltrate, which is expected in reactive germinal centers. However, BCL2 is the most specific marker for follicular lymphoma in this context. Marginal zone lymphomas, while also B-cell lymphomas, often arise from the marginal zone of lymphoid follicles or extranodal lymphoid tissue. They can exhibit varying immunophenotypes, but BCL2 positivity is less consistently observed in the neoplastic cells compared to follicular lymphoma. CD5 is typically negative in both follicular and marginal zone lymphomas, but can be seen in mantle cell lymphoma, which is another differential. CD30 is generally negative in low-grade lymphomas like follicular and marginal zone lymphomas, but can be positive in anaplastic large cell lymphoma or Hodgkin lymphoma, which are less likely given the described morphology. Therefore, the presence of BCL2 positivity in a significant proportion of the lymphoid cells, particularly within the follicular structures, strongly supports a diagnosis of follicular lymphoma and is the most critical ancillary finding among the options provided for differentiating it from other B-cell lymphomas that might present with similar morphology. The explanation focuses on the rationale behind selecting BCL2 as the most discriminative marker in this specific diagnostic context, emphasizing its role in identifying the neoplastic B-cell population characteristic of follicular lymphoma.
-
Question 25 of 29
25. Question
Consider a skin biopsy from the forearm of an elderly patient presenting with a persistent, indurated plaque. Histopathological examination reveals a dense dermal infiltrate composed of atypical lymphoid cells with irregular nuclear contours and scant cytoplasm, predominantly located in the superficial and mid-dermis. Immunohistochemical stains are performed. The lymphoid infiltrate is positive for CD20 and CD79a, confirming B-cell lineage. CD5 staining shows focal positivity within the infiltrate. To definitively establish a neoplastic B-cell process and guide further management, which additional immunohistochemical finding would be most critical for the dermatopathologist at American Board of Pathology – Subspecialty in Dermatopathology University to identify?
Correct
The question probes the understanding of the interplay between specific immunohistochemical markers and the diagnostic classification of cutaneous lymphoid proliferations, a core competency in dermatopathology. The scenario describes a lesion with features suggestive of a B-cell lymphoma, but with atypical morphology. The key to differentiating between a reactive process and a true neoplasm, particularly in challenging cases, lies in the precise interpretation of immunophenotypic profiles. In this case, the observed positivity for CD20 and CD79a confirms a B-cell lineage. The crucial finding for malignancy, especially in the context of atypical morphology, is the presence of clonality. While CD5 is typically associated with T-cell lymphomas and some B-cell lymphomas like CLL/SLL, its presence in a cutaneous B-cell lymphoma is not inherently diagnostic of malignancy but can be a feature of certain subtypes. However, the definitive marker of neoplastic B-cell proliferation, indicating a monoclonal expansion, is the demonstration of kappa or lambda light chain restriction. If the neoplastic B-cells predominantly express kappa light chains, then kappa light chain positivity and lambda light chain negativity would be expected. Conversely, lambda restriction would manifest as lambda positivity and kappa negativity. The absence of significant light chain restriction, or a near equal distribution of both kappa and lambda light chains, would strongly suggest a reactive B-cell process. Therefore, the presence of kappa light chain restriction, alongside the other B-cell markers, solidifies the diagnosis of a neoplastic B-cell proliferation, likely a cutaneous B-cell lymphoma.
Incorrect
The question probes the understanding of the interplay between specific immunohistochemical markers and the diagnostic classification of cutaneous lymphoid proliferations, a core competency in dermatopathology. The scenario describes a lesion with features suggestive of a B-cell lymphoma, but with atypical morphology. The key to differentiating between a reactive process and a true neoplasm, particularly in challenging cases, lies in the precise interpretation of immunophenotypic profiles. In this case, the observed positivity for CD20 and CD79a confirms a B-cell lineage. The crucial finding for malignancy, especially in the context of atypical morphology, is the presence of clonality. While CD5 is typically associated with T-cell lymphomas and some B-cell lymphomas like CLL/SLL, its presence in a cutaneous B-cell lymphoma is not inherently diagnostic of malignancy but can be a feature of certain subtypes. However, the definitive marker of neoplastic B-cell proliferation, indicating a monoclonal expansion, is the demonstration of kappa or lambda light chain restriction. If the neoplastic B-cells predominantly express kappa light chains, then kappa light chain positivity and lambda light chain negativity would be expected. Conversely, lambda restriction would manifest as lambda positivity and kappa negativity. The absence of significant light chain restriction, or a near equal distribution of both kappa and lambda light chains, would strongly suggest a reactive B-cell process. Therefore, the presence of kappa light chain restriction, alongside the other B-cell markers, solidifies the diagnosis of a neoplastic B-cell proliferation, likely a cutaneous B-cell lymphoma.
-
Question 26 of 29
26. Question
A 68-year-old male presents with a pigmented lesion on his left forearm that has been present for several years and has recently shown changes in color and size. A shave biopsy is performed. Histopathological examination reveals a proliferation of atypical melanocytes within the epidermis and extending into the superficial dermis. The lesion demonstrates a Breslow depth of 0.7 mm. Mitotic figures are identified at a rate of 2 per square millimeter in the dermal component, and focal ulceration is noted at the epidermal surface. Considering the principles of dermatopathology as taught at American Board of Pathology – Subspecialty in Dermatopathology University, which combination of factors would be most crucial for accurate staging and predicting the patient’s prognosis?
Correct
The scenario describes a patient with a history of chronic sun exposure and a lesion that exhibits atypical melanocytic proliferation. The question probes the understanding of prognostic indicators in melanoma, specifically focusing on the role of mitotic activity and ulceration in determining the Breslow depth and overall prognosis. While Breslow depth is a primary determinant, the presence of ulceration and mitotic rate are critical adjuncts that significantly impact staging and management decisions, even within a given Breslow depth. For instance, a melanoma with a Breslow depth of 0.8 mm that is ulcerated and has a high mitotic rate would be staged higher than one without these features. The concept of “clark level” is largely superseded by Breslow depth for staging but still informs understanding of invasion. Lymphovascular invasion is a separate, critical prognostic factor that is not directly assessed by the provided histological description. Therefore, the most accurate assessment of the lesion’s aggressive potential, considering the described histological findings and the need for accurate staging for treatment planning at an institution like American Board of Pathology – Subspecialty in Dermatopathology University, would involve a comprehensive evaluation of Breslow depth, ulceration, and mitotic rate. The question requires integrating these factors to determine the most impactful prognostic information.
Incorrect
The scenario describes a patient with a history of chronic sun exposure and a lesion that exhibits atypical melanocytic proliferation. The question probes the understanding of prognostic indicators in melanoma, specifically focusing on the role of mitotic activity and ulceration in determining the Breslow depth and overall prognosis. While Breslow depth is a primary determinant, the presence of ulceration and mitotic rate are critical adjuncts that significantly impact staging and management decisions, even within a given Breslow depth. For instance, a melanoma with a Breslow depth of 0.8 mm that is ulcerated and has a high mitotic rate would be staged higher than one without these features. The concept of “clark level” is largely superseded by Breslow depth for staging but still informs understanding of invasion. Lymphovascular invasion is a separate, critical prognostic factor that is not directly assessed by the provided histological description. Therefore, the most accurate assessment of the lesion’s aggressive potential, considering the described histological findings and the need for accurate staging for treatment planning at an institution like American Board of Pathology – Subspecialty in Dermatopathology University, would involve a comprehensive evaluation of Breslow depth, ulceration, and mitotic rate. The question requires integrating these factors to determine the most impactful prognostic information.
-
Question 27 of 29
27. Question
A dermatopathologist at the American Board of Pathology – Subspecialty in Dermatopathology University is evaluating a skin biopsy from a 65-year-old male presenting with a slowly growing, indurated plaque on the forearm. Histologically, the dermis shows a nodular and diffuse infiltrate of small to medium-sized lymphoid cells with scattered larger atypical cells. Within the infiltrate, there are identifiable germinal centers surrounded by a mantle of lymphocytes and interspersed T-cells. Immunohistochemical stains are performed. Which combination of immunohistochemical findings would most strongly support a diagnosis of a germinal center-derived B-cell lymphoma, such as follicular lymphoma, over other B-cell lymphoproliferative disorders?
Correct
The question probes the understanding of the interplay between specific immunohistochemical markers and the diagnostic classification of cutaneous lymphoid neoplasms, a core competency in dermatopathology. The scenario describes a lesion with features suggestive of a primary cutaneous B-cell lymphoma, specifically a follicular lymphoma or a marginal zone lymphoma, given the presence of germinal centers and reactive T-cells. The key to differentiating these entities often lies in assessing the clonality of the B-cells and their specific immunophenotype. In this context, the expression of CD10 and BCL6 is characteristic of germinal center B-cells, which are the precursors for follicular lymphoma. While BCL6 can also be seen in other B-cell lymphomas, its co-expression with CD10 strongly favors a germinal center origin. Conversely, CD5 is typically associated with mantle cell lymphoma and chronic lymphocytic leukemia/small lymphocytic lymphoma, which are generally not follicular or marginal zone lymphomas. CD21 and CD23 are markers of follicular dendritic cells and can be present in reactive germinal centers, but they are not primary discriminators of B-cell lymphoma subtypes in the same way as CD10 and BCL6. Therefore, a strong positive staining for CD10 and BCL6, coupled with a negative or weak staining for CD5, would most strongly support a diagnosis related to germinal center B-cells, such as follicular lymphoma, or potentially a marginal zone lymphoma with germinal center involvement, which shares some immunophenotypic overlap. The absence of these markers would necessitate consideration of other differential diagnoses.
Incorrect
The question probes the understanding of the interplay between specific immunohistochemical markers and the diagnostic classification of cutaneous lymphoid neoplasms, a core competency in dermatopathology. The scenario describes a lesion with features suggestive of a primary cutaneous B-cell lymphoma, specifically a follicular lymphoma or a marginal zone lymphoma, given the presence of germinal centers and reactive T-cells. The key to differentiating these entities often lies in assessing the clonality of the B-cells and their specific immunophenotype. In this context, the expression of CD10 and BCL6 is characteristic of germinal center B-cells, which are the precursors for follicular lymphoma. While BCL6 can also be seen in other B-cell lymphomas, its co-expression with CD10 strongly favors a germinal center origin. Conversely, CD5 is typically associated with mantle cell lymphoma and chronic lymphocytic leukemia/small lymphocytic lymphoma, which are generally not follicular or marginal zone lymphomas. CD21 and CD23 are markers of follicular dendritic cells and can be present in reactive germinal centers, but they are not primary discriminators of B-cell lymphoma subtypes in the same way as CD10 and BCL6. Therefore, a strong positive staining for CD10 and BCL6, coupled with a negative or weak staining for CD5, would most strongly support a diagnosis related to germinal center B-cells, such as follicular lymphoma, or potentially a marginal zone lymphoma with germinal center involvement, which shares some immunophenotypic overlap. The absence of these markers would necessitate consideration of other differential diagnoses.
-
Question 28 of 29
28. Question
Consider a 68-year-old individual presenting with a slowly growing, erythematous, slightly indurated plaque on the left forearm. Histopathological examination reveals a dense, nodular and diffuse infiltrate of lymphoid cells within the dermis, with some evidence of follicular colonization and occasional germinal center-like structures. Immunohistochemical staining demonstrates that the neoplastic lymphocytes are positive for CD20, BCL2, and MUM1. Crucially, the cells are negative for CD10 but show focal positivity for BCL6. Given these findings, which of the following diagnoses is most strongly supported by the immunophenotype in the context of the provided histological features, aligning with the diagnostic precision expected at the American Board of Pathology – Subspecialty in Dermatopathology University?
Correct
The question probes the understanding of the interplay between specific immunohistochemical markers and the diagnostic classification of cutaneous lymphoid neoplasms, a core competency in dermatopathology. The scenario describes a lesion with features suggestive of a cutaneous B-cell lymphoma, but with atypical morphology. The key to differentiating between a primary cutaneous follicle center lymphoma (PCFCL) and a marginal zone lymphoma (PCMZL) lies in the immunophenotypic profile, particularly the expression of CD10 and BCL6. PCFCL typically exhibits strong positivity for CD10 and BCL6, reflecting their origin from germinal center B-cells. In contrast, PCMZL, originating from extranodal marginal zone B-cells, usually demonstrates negativity for CD10 and variable expression of BCL6. The provided immunopanel shows positivity for CD20, BCL2, and MUM1, which are common to both entities. However, the crucial differentiator is the absence of CD10 expression. While BCL6 is positive, the combination of CD10 negativity with BCL2 and MUM1 positivity, in the context of a follicular-patterned infiltrate, strongly favors a diagnosis of primary cutaneous marginal zone lymphoma over a follicle center lymphoma, which would typically be CD10 positive. Therefore, the immunophenotype described is most consistent with primary cutaneous marginal zone lymphoma. This understanding is critical for accurate diagnosis and subsequent patient management, aligning with the rigorous standards of the American Board of Pathology – Subspecialty in Dermatopathology University, which emphasizes precise diagnostic classification based on integrated clinicopathologic and molecular data.
Incorrect
The question probes the understanding of the interplay between specific immunohistochemical markers and the diagnostic classification of cutaneous lymphoid neoplasms, a core competency in dermatopathology. The scenario describes a lesion with features suggestive of a cutaneous B-cell lymphoma, but with atypical morphology. The key to differentiating between a primary cutaneous follicle center lymphoma (PCFCL) and a marginal zone lymphoma (PCMZL) lies in the immunophenotypic profile, particularly the expression of CD10 and BCL6. PCFCL typically exhibits strong positivity for CD10 and BCL6, reflecting their origin from germinal center B-cells. In contrast, PCMZL, originating from extranodal marginal zone B-cells, usually demonstrates negativity for CD10 and variable expression of BCL6. The provided immunopanel shows positivity for CD20, BCL2, and MUM1, which are common to both entities. However, the crucial differentiator is the absence of CD10 expression. While BCL6 is positive, the combination of CD10 negativity with BCL2 and MUM1 positivity, in the context of a follicular-patterned infiltrate, strongly favors a diagnosis of primary cutaneous marginal zone lymphoma over a follicle center lymphoma, which would typically be CD10 positive. Therefore, the immunophenotype described is most consistent with primary cutaneous marginal zone lymphoma. This understanding is critical for accurate diagnosis and subsequent patient management, aligning with the rigorous standards of the American Board of Pathology – Subspecialty in Dermatopathology University, which emphasizes precise diagnostic classification based on integrated clinicopathologic and molecular data.
-
Question 29 of 29
29. Question
A 72-year-old male, a retired farmer with a significant history of cumulative sun exposure, presents with a firm, erythematous, crusted papule on his left earlobe that has been growing over the past six months. A shave biopsy is performed. Histopathological examination reveals a malignant proliferation of atypical keratinocytes exhibiting marked nuclear pleomorphism, hyperchromasia, irregular chromatin clumping, and abundant eosinophilic cytoplasm. Dyskeratosis is noted in scattered individual cells and small nests. The tumor demonstrates endophytic growth with invasion into the superficial dermis. Crucially, the neoplastic cells are observed to be intimately associated with and encasing small nerve fibers within the dermis. The biopsy margins are free of significant atypic keratinocytes. Considering the implications of these findings for patient management and the emphasis on precision in dermatopathology at American Board of Pathology – Subspecialty in Dermatopathology University, what is the most critical immediate management consideration?
Correct
The scenario describes a patient with a history of chronic sun exposure and a new, rapidly growing lesion. The histological findings of atypical keratinocytes with enlarged, pleomorphic, hyperchromatic nuclei, dyskeratosis, and parakeratosis, along with invasion into the dermis, are classic for squamous cell carcinoma. The presence of perineural invasion, as indicated by the tumor cells surrounding nerve fibers, is a critical prognostic factor that significantly increases the risk of local recurrence and metastasis. Therefore, the most appropriate next step in management, reflecting the rigorous standards of dermatopathology training at American Board of Pathology – Subspecialty in Dermatopathology University, is to ensure adequate surgical margins during excision to encompass the extent of the tumor and any microscopic spread, particularly along the perineural pathways. This proactive approach minimizes the chance of residual disease and improves patient outcomes. Other options, while potentially relevant in other contexts, do not directly address the immediate management imperative posed by the perineural invasion in this high-risk squamous cell carcinoma. Sentinel lymph node biopsy is typically reserved for melanomas with specific depth criteria or high-risk squamous cell carcinomas with clinical nodal involvement, which is not explicitly stated here. Topical chemotherapy or cryotherapy are generally for actinic keratoses or very superficial squamous cell carcinomas, not for invasive lesions with perineural involvement. Radiation therapy might be considered as an adjuvant treatment post-operatively if margins are positive or in cases of unresectable disease, but initial surgical management with clear margins is the primary goal.
Incorrect
The scenario describes a patient with a history of chronic sun exposure and a new, rapidly growing lesion. The histological findings of atypical keratinocytes with enlarged, pleomorphic, hyperchromatic nuclei, dyskeratosis, and parakeratosis, along with invasion into the dermis, are classic for squamous cell carcinoma. The presence of perineural invasion, as indicated by the tumor cells surrounding nerve fibers, is a critical prognostic factor that significantly increases the risk of local recurrence and metastasis. Therefore, the most appropriate next step in management, reflecting the rigorous standards of dermatopathology training at American Board of Pathology – Subspecialty in Dermatopathology University, is to ensure adequate surgical margins during excision to encompass the extent of the tumor and any microscopic spread, particularly along the perineural pathways. This proactive approach minimizes the chance of residual disease and improves patient outcomes. Other options, while potentially relevant in other contexts, do not directly address the immediate management imperative posed by the perineural invasion in this high-risk squamous cell carcinoma. Sentinel lymph node biopsy is typically reserved for melanomas with specific depth criteria or high-risk squamous cell carcinomas with clinical nodal involvement, which is not explicitly stated here. Topical chemotherapy or cryotherapy are generally for actinic keratoses or very superficial squamous cell carcinomas, not for invasive lesions with perineural involvement. Radiation therapy might be considered as an adjuvant treatment post-operatively if margins are positive or in cases of unresectable disease, but initial surgical management with clear margins is the primary goal.